Exam 2 Questions

Réussis tes devoirs et examens dès maintenant avec Quizwiz!

You are screening people at the mall as part of a health fair. The first person who comes for screening has a blood pressure of 132/85. How would you categorize this? A) Normal B) Prehypertension C) Stage 1 hypertension D) Stage 2 hypertension

Prehypertension Prehypertension is considered to be a systolic blood pressure from 120 to 139 and a diastolic BP from 80 to 89. Previously, this was considered normal. JNC 7 recommends taking action at this point to prevent worsening hypertension. Research shows that this population is likely to progress to more serious stages of hypertension.

Otherwise healthy 28yo lawyer presents to ED with 1 day hx of severe abdominal pain. ED doctor suspects appendicitis & general surgery consulted. The resident believes pt has s/s peritonitis on exam, which of the following supports this?

Pressing down onto abdomen firmly & slowly & withdrawing hand quickly produces pain Rebound tenderness, along with guarding & rigidity suggest peritonitis

Mrs. Adams would like to begin an exercise program and was told to exercise as intensely as necessary to obtain a heart rate 60% or greater of her maximum heart rate. She is 52. What heart rate should she achieve? A) 80 B) 100 C) 120 D) 140

100 Maximum heart rate is calculated by subtracting the patient's age from 220. For Mrs. Adams, 60% of this number is about 100. She must also be instructed in how to measure her own pulse or have a device to do so. Most people are able to carry on a conversation at this level of exertion.

Asymmetric BPs are seen in which of the following conditions? A) Coronary artery disease B) Congenital narrowing of the aorta C) Diffuse atherosclerosis D) Vasculitis, as seen in systemic lupus erythematosus

Congenital narrowing of the aorta A difference of as little as 10 mm Hg in the systolic blood pressure may be significant. Coarctation and dissecting aortic aneurysm are causes of asymmetric blood pressures. Coarctation represents a congenital narrowing of the aorta. While some forms of vasculitis can affect large vessels where we measure the blood pressure, lupus is generally a small-vessel vasculitis. Usually, neither diffuse atherosclerosis nor coronary artery disease is responsible for a focal difference in blood pressure.

Otherwise health 31yo accoutant presents to outpatient clinic with 3 yr hx of recurrent crampy abdominal pain that lasts for 1-2 weeks each episode & is associated with onset of constipation, she describes infrequent, small hard stool that she finds very difficult to pass. She has tried to increase dietary fiber & water intake, but usually this is not sufficient & she reports to OTC laxatives, which she finds upset her stomach, but do resolve the constipation. S/s typically gradually resolve with BMs. Which of the following is most likely physiological mechanism for constipation?

Functional change in BM Characteristic of IBS

You are participating in a health fair and performing cholesterol screens. One person has a cholesterol of 225. She is concerned about her risk for developing heart disease. Which of the following factors is used to estimate the 10-year risk of developing coronary heart disease? A) Ethnicity B) Alcohol intake C) Gender D) Asthma

Gender Gender is used in the calculation of the 10-year risk for developing coronary heart disease, because men have a higher risk than women.

Which of the following correlates with a sustained, high-amplitude PMI? A) Hyperthyroidism B) Anemia C) Fever D) Hypertension

Hypertension While hyperthyroidism, anemia, and fever can cause a high-amplitude PMI, pressure work by the heart, as seen in hypertension, causes the PMI to be sustained.

The ankle-brachial index (ABI) is calculated by dividing the systolic BP at the dorsalis pedis by the systolic BP at the brachial artery. Which of the following values would be consistent with mild peripheral arterial disease? A) 1.1 B) 0.85 C) 0.65 D) 0.35

0.85 The mild disease is represented by an ABI of 0.71 to 0.9. Any value above 0.9 is normal. Moderate disease is defined as between 0.7 and 0.41, and severe disease is defined as 0.4 or less. Patients in the "severe" category have a 20% to 25% annual risk of death.

55F sustains heart attack & fu EKG shows LBBB, what would be the duration of QRS?

125ms Duration of ventricular depolarization & normally < 100ms

On auscultation of the chest, a split S2 is best heard at: A) Second intercostal space, right sternal border B) Second intercostal space, left sternal border C) Fifth intercostal space, midclavicular line D) Fourth intercostal space, left sternal border

316. B) Second intercostal space, left sternal border The S2 heart sound results from closure of the aortic and pulmonic valves. The right ventricular systolic ejection time is longer than the left, so the pulmonic valve closes slightly later than the aortic valve. The slightly different closing times of the valves make the S2 heart sound. It is heard best using the diaphragm of the stethoscope.

You are assessing a patient for peripheral vascular disease in the arms, secondary to a complaint of increased weakness and a history of coronary artery disease and diabetes. You assess the brachial and radial pulses and note that they are bounding. What does that translate to on a scale of 0 to 3? A) 0 B) 3+ C) 2+ D) 1+

3+ A pulse of 3+ is considered to be bounding.

45 physician placed on BB for HTN, prior to med administration, pt HR 75 with CO of 5lpm, following giving med, HR decreases to 60bpm without change in stroke volume, expected new cardiac output?

4lpm product of HR x SV

A 51-year-old cook comes to your office for consultation. She recently found out that her 44-year-old sister with premenopausal breast cancer is positive for the BRCA1 gene. Your patient has been doing research on the Internet and saw that her chance of having also inherited the BRCA1 gene is 50%. She is interested in knowing what her risk of developing breast cancer would be if she were positive for the gene. She denies any lumps in her breasts and has had normal mammograms. She has had no weight loss, fever, or night sweats. Her mother is healthy and her father has prostate cancer. Two of her paternal aunts died of breast cancer. She is married. She denies using tobacco or illegal drugs and rarely drinks alcohol. Her breast and axilla examinations are unremarkable. At her age, what is her risk of getting breast cancer if she has the BRCA1 gene? A) 10% B) 50% C) 80%

80% At the age of 50, the risk of breast cancer for someone with the BRCA1 gene is 50%.

20. Which of the following is true regarding breast self-examination? A) It has been shown to reduce mortality from breast cancer. B) It is recommended unanimously by organizations making screening recommendations C) A high proportion of breast masses are detected by breast self-examination. D) The undue fear caused by finding a mass justifies omitting instruction in breast self-examination.

A high proportion of breast masses are detected by BSE. Although self-examination has not been shown to reduce mortality and is not recommended by all groups making screening recommendations, many choose to teach women a systematic method in which to examine their breasts. A high proportion of breast masses are detected by breast self-examination.

The most current Eighth Joint National Committee (JNC 8) recommendation for the blood pressure goal in diabetics is: A) <140/90 mmHg B) <130/85 mmHg C) <130/80 mmHg D) <125/75 mmHg

A) <140/90 mmHg The Eighth Joint National Committee (JNC 8) blood pressure goal for adults and older adults 60 years of age or older who have diabetes (with or without chronic kidney disease) is blood pressure less than 140/90. In addition, all patients with hypertension should have lifestyle intervention (i.e., weight loss, smoking cessation, healthy diet), which should be continued throughout treatment.

According to the guidelines outlined in the Eighth Joint National Committee (JNC 8) the target blood pressure goal for hypertensive patients who are aged 60 years or older is: A) <150/90 mmHg B) <130/85 mmHg C) <120/80 mmHg D) <110/75 mmHg

A) <150/90 mmHg For the general population (no diabetes or chronic kidney disease) the recommended systolic blood pressure (SBP) goal at age 60 years or older goes up by 10 mmHg to blood pressure less than 150/But if the patient is a diabetic and/or has chronic kidney disease, then the blood pressure goal does not change and continues at blood pressure less than 140/90.

An 80-year-old man with hypertension and hyperlipidemia presents with complaints of the rapid onset of severe low-back pain accompanied by abdominal pain that is gradually worsening. The patient appears pale and complains that he does not feel well. During the abdominal exam, the nurse practitioner detects a soft pulsatile mass just above the umbilicus as she palpates this area with her hand. Which of the following conditions is most likely? A) Abdominal aortic aneurysm B) Cauda equina syndrome C) Acute diverticulitis D) Adenocarcinoma of the colon

A) Abdominal aortic aneurysm Elderly males who are ex-smokers are at higher risk for abdominal aortic aneurysm. The aneurysm is usually asymptomatic and is discovered incidentally during a routine chest x-ray or abdominal ultrasound. Although small aneurysms are usually not detectable during abdominal exams, the larger aneurysms may be palpable during an abdominal exam, but abdominal obesity will obscure the findings. The symptoms in this case point toward a rapidly dissecting aneurysm. The best action is to call 911 stat.

A positive obturator sign might signify which of the following conditions? A) Acute appendicitis B) Acute pancreatitis C) Acute cholecystitis D) Acute hepatitis

A) Acute appendicitis A positive obturator sign may indicate acute appendicitis. The test is performed with the patient supine. The examiner rotates the hip, using full range of motion. The test is positive if pain is experienced with movement or flexion of the hip.

Cullen's sign is most commonly associated with which of the following? A) Acute pancreatitis B) Myocardial infarction C) Acute pyelonephritis D) Preeclampsia

A) Acute pancreatitis Cullen's sign is commonly seen in acute pancreatitis. It refers to a yellowish-blue skin color change around the umbilicus. It is thought to occur due to the pancreatic enzymes that run along the ligament and subcutaneous tissues around the umbilicus.

Grey-Turner's sign is highly suggestive of which of the following conditions? A) Acute pancreatitis B) Acute appendicitis C) Acute diverticulitis D) Gastric cancer

A) Acute pancreatitis Grey-Turner's sign is the acute onset of bluish discoloration located on the flank area that is caused by bruising. It is usually associated with severe acute pancreatitis, but it can also be found in some cases of ruptured ectopic pregnancy.

All of the following are possible causes for secondary hypertension except: A) Acute pyelonephritisB) PheochromocytomaC) Renovascular stenosis D) Coarctation of the aorta

A) Acute pyelonephritis Acute pyelonephritis does not involve any vascular change; however, chronic pyelonephritis (reflux nephropathy) is a factor for secondary hypertension. Pheochromocytoma is a rare tumor of the adrenal glands that results in a release of too much epinephrine and norepinephrine, hormones that control heart rate, metabolism, and blood pressure. Renovascular stenosis is a narrowing of one or both arteries leading to the kidneys. It can cause severe hypertension and irreversible kidney damage. Coarctation of the aorta is a congenital heart defect of the aorta; it is a narrowing of the aorta that causes the heart to work harder to get blood to flow through the narrow aortic passageway to other organs, which, in turn, causes an increase in blood pressure.

Rovsing's sign is associated with which of the following? A) An acute abdomen, such as during a ruptured appendix B) Knee instability C) Damage to the meniscus of the knee D) Acute cholelithiasis

A) An acute abdomen, such as during a ruptured appendix Rovsing's sign identifies an acute abdomen, such as occurs in acute appendicitis. With the patient in the supine position, the examiner palpates deep into the left lower quadrant of the abdomen. The maneuver is positive if pain is referred to the right lower quadrant.

A 55-year-old male patient describes an episode of chest tightness in his substernal area that radiated to his back while he was jogging. It was relieved immediately when he stopped. The patient's symptoms are highly suggestive of what condition? A) Angina pectoris B) Acute myocardial infarction C) Gastroesophageal reflux disease D) Acute costochondritis

A) Angina pectoris The classic pain of angina is described as discomfort, pressure, tightness, or heaviness on the center or left side of the chest that is precipitated by exertion and relieved by rest. The pain can be referred to the back, shoulders, neck, or jaw. The pain is not sharp, knife-like, or stabbing in nature. Angina is caused by transient myocardial ischemia. The most common cause of angina is coronary artery disease. If the angina worsens, is not relieved by rest, or lasts more than 20 minutes, it may be due to acute myocardial infarction (MI). If acute MI is suspected, call 911.

A medium-pitched harsh mid-systolic murmur is best heard at the right second intercostal space of the chest. It radiates into the neck. Which of the following is the correct diagnosis? A) Aortic stenosis B) Pulmonic stenosis C) Aortic regurgitation D) Mitral stenosis

A) Aortic stenosis The murmur associated with aortic stenosis can be auscultated as harsh and high pitched in the right second intercostal space. It typically radiates to the carotid arteries and apex.

You can determine a pulse deficit by counting the: A) Apical and radial pulses at the same time, then finding the difference between the two B) Apical pulse first, then the radial pulse, and subtracting to find the difference between the two C) Apical pulse and the femoral pulse at the same time and finding the difference between the two D) Radial pulse first, then counting the femoral pulse, and subtracting to find the difference between the two

A) Apical and radial pulses at the same time, then finding the difference between the two The pulse deficit is the difference between the apical pulse and the radial pulse. These should be taken at the same time, which will require that two people take the pulse: one with a stethoscope and one at the wrist. Count for 1 full minute. Then subtract the radial from the apical.

While doing a cardiac exam on a 45-year-old man, you note an irregular rhythm with a pulse rate of 110 beats/min. The patient is alert and is not in distress. What is the most likely diagnosis? A) Atrial fibrillation B) Ventricular fibrillation C) Cardiac arrhythmia D) First-degree right bundle branch block

A) Atrial fibrillation Atrial fibrillation is chaotic electrical activity of the heart, caused by several ectopic foci in the atria without any signs of distress. There are three pathological irregular rhythms: (1) ectopic beats (may be atrial, junctional, or ventricular), (2) atrial fibrillation, and (3) second-degree heart block. All are confirmed by EKG.

Which of the following is a true statement regarding acute gastritis? A) Chronic intake of nonsteroidal anti-inflammatory drugs (NSAIDs) can cause the disorder B) Chronic lack of dietary fiber is the main cause of the disorder C) The screening test for the disorder is the barium swallow test D) The gold standard to evaluate the disorder is a colonoscopy

A) Chronic intake of nonsteroidal anti-inflammatory drugs (NSAIDs) can cause the disorder Signs and symptoms of gastritis are nausea/vomiting, upset stomach, loss of appetite, and burning/aching or gnawing pain located in the epigastric area. Nonselective NSAIDs (aspirin, ibuprofen, naproxen, others) have adverse effects on the gastrointestinal (GI) tract, kidneys, central nervous system, and cardiovascular effects, and decrease platelet aggregation (aspirin). Chronic use of nonselective NSAIDs disrupts the production of prostaglandins, which involves cycloxygenase-1 (COX-1) and COX-2. The GI mucosa uses COX-1 to produce mucosal protective factors. Blocking COX-1 decreases these protective factors and increases risk of gastritis, ulcers, and GI bleeding. Selective NSAIDs, such as celecoxib (Celebrex), do less damage to the GI tract because they block only COX-2, which is responsible for pain and inflammation.

An adult patient who was recently discharged from the hospital was prescribed clindamycin. The patient reports that he took his last dose yesterday. He presents in the primary care clinic with a complaint of recent onset of watery diarrhea from 10 to 15 times a day with abdominal cramping. He denies fever and chills. Which of the following conditions is most likely in this patient? A) Clostridium difficile-associated diarrhea (CDAD) B) GiardiasisC) Pseudomembranous colitisD) Irritable bowel syndrome

A) Clostridium difficile-associated diarrhea (CDAD) Important risk factors for CDAD and C. difficile colitis are antibiotic therapy and hospitalization. Almost any antibiotic can cause the condition, but the most common are clindamycin, cephalosporins, and fluoroquinolones. Diarrhea can occur during as well as after therapy (5-10 days; up to 10 weeks). Pseudomembranous colitis is a complication of C. difficile colitis.

An adult patient was recently discharged from the hospital with a prescription 597 of clindamycin. The patient reports that he took his last dose yesterday. He presents in the primary care clinic with complaints of the recent onset of watery diarrhea from 10 to 15 times a day with abdominal cramping. He denies fever and chills. Which of the following conditions is most likely in this patient? A) Clostridium difficile-associated diarrhea B) GiardiasisC) Pseudomembranous colitisD) Irritable bowel syndrome

A) Clostridium difficile-associated diarrhea An important risk factor for 699 Clostridium difficile-associated diarrhea (CDAD) and C. difficile colitis is antibiotic therapy and hospitalization. Almost any antibiotic can cause the condition, but the most common are clindamycin, cephalosporins, and fluoroquinolones. Diarrhea can occur during therapy as well as after therapy (5-10 days; up to 10 weeks). Pseudomembranous colitis is a complication of C. difficile colitis.

The S1 heart sound is caused by: A) Closure of the atrioventricular valves B) Closure of the semilunar valves C) Opening of the atrioventricular valves D) Opening of the semilunar valves

A) Closure of the atrioventricular valves A heart valve normally allows blood to flow in only one direction. A heart valve opens or closes incumbent upon differential blood pressure on each side. A form of heart disease occurs when a valve malfunctions and allows some blood to flow in the wrong direction. The S1 heart sound is caused by turbulence caused by the closure of mitral and tricuspid valves at the start of systole.

A nurse practitioner is taking part in a community outreach program for a local hospital. Most of her audience has a diagnosis of hypertension. They are all interested in learning more about a proper diet. When discussing potential sources of potassium and magnesium, which of the following are the best sources for these two minerals? A) Fruits, leafy greens, and nuts B) Whole grains, red meat, and dairy C) Bananas, beef, and yogurt D) Mushrooms, fermented foods, and vegetables

A) Fruits, leafy greens, and nuts The best sources of both potassium and magnesium are fruits (bananas, cantaloupe, papaya, kiwi), leafy greens (spinach, kale, chard), and nuts (cashews, almonds, walnuts, peanuts).

The nurse practitioner is educating a new patient with Raynaud's phenomenon about lifestyle recommendations to decrease exacerbations of the disorder. Which of the following lifestyle changes would not be useful for this patient? A) Increasing consumption of caffeine-containing drinks and foods such as chocolate B) C) D) Wearing gloves or mittens during cold weather and being careful when handling frozen foodsSmoking and exercising at least three times a weekDecreasing emotional stress and lifestyle stressors

A) Increasing consumption of caffeine-containing drinks and foods such as 695 chocolate Lifestyle changes associated with decreasing exacerbations are wearing gloves or mittens during cold weather, taking care when handling frozen foods (wear gloves), and avoiding vasoconstricting agents (caffeine, smoking, cocaine, amphetamines) and emotional stress. Exercise and reducing lifestyle stress are recommended. Raynaud's disease usually involves the fingers and/or toes due to severe arteriolar vasospasm causing ischemia. During an exacerbation, the fingers change color becoming white, blue, and red (think of the American flag as a reminder). Raynaud's is classified either as primary (Raynaud's disease) or secondary (Raynaud's phenomenon). Individuals with secondary Raynaud's have a higher risk of autoimmune disorders such as scleroderma, Sjögren's syndrome, and systemic lupus erythematosus. The disorder affects mostly young women (ages 15-30 years).

A 20-year-old woman reports that for several years, she has had random episodes of palpitations and shortness of breath that resolve spontaneously. She denies chest pain, arm pain, and syncope. Her past medical and family histories are negative for coronary artery disease, stroke, or lung disease. During the cardiac exam, the nurse practitioner notices a grade 3/6 murmur that is accompanied by a mid-systolic click, which is heard best heard at the apical area. The apical pulse is 78 beats/min, blood pressure is 120/60 mmHg, and temperature is 98.6°F. The cardiac exam is highly suggestive of which of the following conditions? A) Mitral valve prolapse B) Aortic stenosis C) Atrial septal defect D) Pulmonary regurgitation

A) Mitral valve prolapse A systolic murmur that is accompanied by a mid systolic click located at the apical area is a classic finding of mitral valve prolapse (MVP). Most cases of MVP are asymptomatic. To detect MVP, order an echocardiogram with Doppler imaging.

An urgent care nurse practitioner is assessing a 45-year-old White woman with a body mass index (BMI) of 32 for a complaint of intermittent right upper quadrant abdominal pain over the past few weeks that is precipitated by eating fried foods and peanut butter. On exam, the patient's heart and lungs are normal. There is no pain over the costovertebral angle. During abdominal exam, bowel sounds are present in all quadrants. While the nurse is palpating deeply on the right upper quadrant during deep inspiration, the patient complains of severe sharp pain. Which of the following is best described? A) Murphy's sign B) McMurray's sign C) Rovsing's sign D) Obturator sign

A) Murphy's sign Murphy's sign is suggestive of acute cholecystitis and gallbladder disease. It is elicited by palpating the subcostal region on the right upper abdomen; in response, the patient abruptly stops inspiration because of the severe pain.

Which of the following is an accurate description of eliciting Murphy's sign? A) On deep inspiration by the patient, palpate firmly in the right upper quadrant of the abdomen below the costovertebral angle B) Bend the patient's hips and knees at 90 degrees, then passively rotate the hip externally, and internally C) Ask the patient to squat, then place the stethoscope on the apical area D) Press into the abdomen deeply, then release it suddenly

A) On deep inspiration by the patient, palpate firmly in the right upper quadrant of the abdomen below the costovertebral angle Murphy's sign is tested during an abdominal examination for biliary disorders. As the patient breathes in, the abdominal contents are pushed downward as the diaphragm moves down and the lungs expand. As the patient stops/hold the breath, the gallbladder comes in contact with the examiner's fingers and may elicit pain. To be considered positive, the same maneuver must not elicit pain when performed on the left side. A negative Murphy's test in the elderly is not useful for ruling out cholecystitis if history and other tests suggest the diagnosis.

Lifestyle modifications are an important aspect in the treatment of 560 hypertension. Which of the following statements is incorrect? A) Reduce intake of sodium, potassium, and calcium B) Reduce intake of sodium and saturated fats C) Exercise at least three to four times per week D) Maintain an adequate intake of potassium, magnesium, and calcium

A) Reduce intake of sodium, potassium, and calcium Lifestyle modifications for hypertension include exercise three to four times a week, diet modifications of reduced intake of sodium and saturated fats, and adequate dietary intake of potassium, magnesium, and calcium.

The bell of the stethoscope is best used for auscultation of which of the following? A) S3 and S4 and low-pitched tones B) S3 and S4 only C) S1 and S2 and high-pitched tones D) S1 and S2 only

A) S3 and S4 and low-pitched tones The bell is most useful for picking up low- pitched sounds; for example, S3, S4, and mitral stenosis. The diaphragm is most useful for picking up high-pitched sounds; for example, S1, S2, aortic or mitral regurgitation, and pericardial friction rubs.

During the physical exam of a 60-year-old adult, the nurse practitioner performs an abdominal exam. The nurse practitioner is checking the left upper quadrant of the abdomen. During percussion, an area of dullness is noted beneath the lower left ribcage. Which of the following is a true statement regarding the spleen? A) The spleen is not palpable in the majority of healthy adults B) The spleen is 8 to 10 cm in the left midaxillary line at its longest axis C) The spleen is 2 to 6 cm between the 9th and 11th ribs on the left midaxillary line D) The splenic size varies depending on the patient's gender

A) The spleen is not palpable in the majority of healthy adults The spleen is located in the left upper quadrant of the abdomen under the diaphragm and is protected by the lower ribcage. In the majority of adults, it is not palpable. The spleen's longest axis is 11 to 20 cm. Any spleen larger than 20 cm is enlarged. The best test for evaluating splenic (or hepatic) size is the abdominal ultrasound. Disorders that can cause splenomegaly include mononucleosis, sickle cell disease, congestive heart failure, bone marrow cancers (myeloma, leukemia), and several other diseases.

A 68-year-old retired truck driver comes to your office for evaluation of swelling in his legs. He is a smoker and has been taking medications to control his hypertension for the past 25 years. You are concerned about his risk for peripheral vascular disease. Which of the following tests are appropriate to order to initially evaluate for this condition? A) Venogram B) CT scan of the lower legs C) Ankle-brachial index (ABI) D) PET scan

ABI The ankle-brachial index is a good test for obtaining information about significant stenosis in the vessels of the lower extremities. Sixteen percent of patients with known peripheral vascular disease also have coronary artery disease.

Mrs. Patton, a 48-year-old woman, comes to your office with a complaint of a breast mass. Without any other information, what is the risk of this mass being cancerous? A) About 10% B) About 20% C) About 30% D) About 40%

About 10% Eleven percent of women presenting with a breast mass will have breast cancer. This statistic can be reassuring to a patient, but the importance of further studies must be emphasized.

A 23-year-old computer programmer comes to your office for an annual examination. She has recently become sexually active and wants to be placed on birth control. Her only complaint is that the skin in her armpits has become darker. She states it looks like dirt, and she scrubs her skin nightly with soap and water but the color stays. Her past medical symptoms consist of acne and mild obesity. Her periods have been irregular for 3 years. Her mother has type 2 diabetes and her father has high blood pressure. The patient denies using tobacco but has four to five drinks on Friday and Saturday nights. She denies any illegal drug use. On examination you see a mildly obese female who is breathing comfortably. Her vital signs are unremarkable. Looking under her axilla, you see dark, velvet-like skin. Her annual examination is otherwise unremarkable. What disorder of the breast or axilla is she most likely to have? A) Peau d'orange B) Acanthosis nigricans C) Hidradenitis suppurativa

Acanthosis nigricans Acanthosis nigricans can be associated with an internal malignancy, but in most cases it is a benign dermatologic condition associated with polycystic ovarian syndrome, consisting of acne, hirsutism, obesity, irregular periods, infertility, ovarian cysts, and early onset type 2 diabetes. It is also known to correlate with insulin resistance.

A 26-year-old sports store manager comes to your clinic, complaining of severe right-sided abdominal pain for 12 hours. He began having a stomachache yesterday, with a decreased appetite, but today the pain seems to be just on the lower right side. He has had some nausea and vomiting but no constipation or diarrhea. His last bowel movement was last night and was normal. He has had no fever or chills. He denies any recent illnesses or injuries. His past medical history is unremarkable. He is engaged. He denies any tobacco or drug use and drinks four to six beers per week. His mother has breast cancer and his father has coronary artery disease. On examination he appears ill and is lying on his right side. His temperature is 100.4 and his heart rate is 110. His bowel sounds are decreased and he has rebound and involuntary guarding, one third of the way between the anterior superior iliac spine and the umbilicus in the right lower quadrant. His rectal, inguinal, prostate, penile, and testicular examinations are normal. What is the most likely cause of his pain? A) Acute appendicitis B) Acute mechanical intestinal obstruction C) Acute cholecystitis D) Mesenteric ischemia

Acute appendicitis Appendicitis is common in the young and usually presents with periumbilical pain that localizes to the right lower quadrant in an area known as McBurney's Point, described above as one third of the way between the anterior superior iliac spine and the umbilicus on the right. Rebound and guarding are common. Remote rebound or Rovsing's sign is also seen commonly when the course of appendicitis is advanced. Bowel movements are usually unaffected.

A 76-year-old retired farmer comes to your office complaining of abdominal pain, constipation, and a low-grade fever for about 3 days. He denies any nausea, vomiting, or diarrhea. The only unusual thing he remembers eating is two bags of popcorn at the movies with his grandson, 3 days before his symptoms began. He denies any other recent illnesses. His past medical history is significant for coronary artery disease and high blood pressure. He has been married for over 50 years. He denies any tobacco, alcohol, or drug use. His mother died of colon cancer and his father had a stroke. On examination he appears his stated age and is in no acute distress. His temperature is 100.9 degrees and his other vital signs are unremarkable. His head, cardiac, and pulmonary examinations are normal. He has normal bowel sounds and is tender over the left lower quadrant. He has no rebound or guarding. His rectal examination is unremarkable and his fecal occult blood test is negative. His prostate is slightly enlarged but his testicular, penile, and inguinal examinations are all normal. Blood work is pending. What diagnosis for abdominal pain best describes his symptoms and signs? A) Acute diverticulitis B) Acute cholecystitis C) Acute appendicitis D) Mesenteric ischemia

Acute diverticulitis Diverticulitis is caused by localized infections within the colonic diverticula. Constipation, fever, and abdominal pain are common. Mesenteric ischemia classically presents in older people with a history of vascular disease elsewhere. The typical pain is unusual in that it is not made worse by examination despite being severe. Some mistake this feature to indicate malingering, with bad results.

A 22-year-old law student comes to your office, complaining of severe abdominal pain radiating to his back. He states it began last night after hours of heavy drinking. He has had abdominal pain and vomiting in the past after drinking but never as bad as this. He cannot keep any food or water down, and these symptoms have been going on for almost 12 hours. He has had no recent illnesses or injuries. His past medical history is unremarkable. He denies smoking or using illegal drugs but admits to drinking 6 to 10 beers per weekend night. He admits that last night he drank something like 14 drinks. On examination you find a young male appearing his stated age in some distress. He is leaning over on the examination table and holding his abdomen with his arms. His blood pressure is 90/60 and his pulse is 120. He is afebrile. His abdominal examination reveals normal bowel sounds, but he is very tender in the left upper quadrant and epigastric area. He has no Murphy's sign or tenderness in the right lower quadrant. The remainder of his abdominal examination is normal. His rectal, prostate, penile, and testicular examinations are normal. He has no inguinal hernias or tenderness with that examination. Blood work is pending. What etiology of abdominal pain is most likely causing his symptoms? A) Peptic ulcer disease B) Biliary colic C) Acute cholecystitis D) Acute pancreatitis

Acute pancreatitis Acute pancreatitis causes epigastric and left upper quadrant pain and often radiates into the back. There is often a history of long-standing gallbladder disease or recent alcohol ingestion. Severe abdominal pain and vomiting are often seen. Medications such as proton pump inhibitors can also cause pancreatitis in people without these other risk factors. Treatment includes hydration, pain management, and bowel rest.

26yo graduate student in physical anthropology presents for routine annual exam, in discussing risks & benefits of BSE, she requests technical definition of area covered by BSE in keeping with her background. Which of the following accurately describes the layout of the human breast?

Against the anterior thoracic wall, from 2nd to 6th rib, and from the sternum to midaxillary line Though some variation exists between women.

To hear a soft murmur or bruit, which of the following may be necessary? A) Asking the patient to hold her breath B) Asking the patient in the next bed to turn down the TV C) Checking your stethoscope for air leaks D) All of the above

All of the above All examiners should carefully search for soft murmurs and bruits. These can have great clinical significance. A quiet patient and room, as well as an intact stethoscope, will greatly increase your ability to hear soft sounds.

Bill, a 55-year-old man, presents with pain in his epigastrium which lasts for 30 minutes or more at a time and has started recently. Which of the following should be considered? A) Peptic ulcer B) Pancreatitis C) Myocardial ischemia D) All of the above

All of the above Epigastric pain can have many causes. History and physical will help discern which causes are most likely, but it is important to realize that any of the above, including myocardial ischemia, is always a possibility. Pneumonia and gallbladder pain can also cause pain in this location.

Which of the following may be missed unless the patient is placed in the left lateral decubitus position and auscultated with the bell? A) Mitral stenosis murmur B) Opening snap of the mitral valve C) S3 and S4 gallops D) All of the above

All of the above Placing the patient in the left lateral decubitus position and auscultating with the bell will enable you to hear these sounds, which would otherwise be missed.

You find a bounding carotid pulse on a 62-year-old patient. Which murmur should you search out? A) Mitral valve prolapse B) Pulmonic stenosis C) Tricuspid insufficiency D) Aortic insufficiency

Aortic insufficiency Bounding carotid pulses would be found in aortic insufficiency. This should be sought by listening over the third left intercostal space, with the patient leaning forward in held exhalation. This is a very soft diastolic murmur usually. A bounding pulse may also be seen in any condition which increases cardiac output, including stimulant use, anxiety, hyperthyroidism, fever, etc.

77M co progressive SOB & dizziness, undergoes cardiac cath, SBP LV 180, aorta is 140, most likely experiencing s/s r/t what valvular condition?

Aortic stenosis Pts often experience dyspnea with exertion, CP or dizziness

You are palpating the apical impulse in a patient with heart disease and find that the amplitude is diffuse and increased. Which of the following conditions could be a potential cause of an increase in the amplitude of the impulse?

Aortic stenosis, with pressure overload of the LV Pressure overload of the left ventricle, such as occurs in aortic stenosis, may result in an increase in amplitude of the apical impulse. The other conditions should decrease amplitude of the apical impulse or not be palpable at all.

Chris is a 20-year-old college student who has had abdominal pain for 3 days. It started at his umbilicus and was associated with nausea and vomiting. He was unable to find a comfortable position. Yesterday, the pain became more severe and constant. Now, he hesitates to walk, because any motion makes the pain much worse. It is localized just medial and inferior to his iliac crest on the right. Which of the following is most likely? A) Peptic ulcer B) Cholecystitis C) Pancreatitis D) Appendicitis

Appendicitis This is a classic history for appendicitis. Notice that the pain has changed from visceral to parietal. It is well localized to the right lower quadrant, making appendicitis a strong consideration.

Diminished radial pulses may be seen in patients with which of the following? A) Aortic insufficiency B) Hyperthyroidism C) Arterial emboli D) Early "warm" septic shock

Arterial emboli Arterial emboli would decrease flow to a region, and therefore pulses would decrease as well. The other conditions actually cause bounding pulses. Aortic insufficiency can cause significant leakage of blood back to the heart, so the heart compensates by increasing forward flow. Stroke volume can increase dramatically with hyperthyroidism, especially in "thyroid storm." This also results in bounding pulses. Although shock generally causes decreased blood pressure and pulses, early septic shock can produce increased peripheral circulation and increase pulses.

46yo executive obese otherwise healthy presents to family medicine clinic with 3 mos course of recurrent severe abdominal pain that usually resolves on its own after few hours, last episode was prolonged lasting 6 hours, frustrated that she has had to leave or miss work on three occasions, she would like a dx & problem fixed, which s/s would be most suggestive of DX of biliary colic?

Associated R-shoulder pain Pain with biliary colic can produce referred pain to right shoulder or scapula due to irritation of right hemidiaphragm

20yo college student experiencing dyspnea on exertion & palpitations, on cardiac auscultation, 2nd heart sound is split & fixed on both inspiration & expiration, what is most likely cardiac condition associated with this finding?

Atrial septal defect Often experience dyspnea as well as atrial arrhythmias

On exam, 40yo teacher has single second heart sound, what is the explanation for this?

Auscultation during expiration During expiration, components of S2, A2, P2 fuse into single sound

A woman who is in the third trimester of pregnancy presents to the nurse practitioner for a physical exam. During the physical exam, the nurse practitioner finds all of the following cardiac changes associated with pregnancy except: A) Systolic ejection murmur B) Diastolic murmur C) Displaced apical impulse D) Louder S1 and S2

B) Diastolic murmur Diastolic murmurs are more likely to be pathological. The heart is displaced in a more transverse position that is lateral to the midclavicular line. The systolic ejection murmur is due to increased stroke volume caused by increased cardiac output and higher basal heart rate.

A 21-year-old woman who is complaining of random palpitations is diagnosed with mitral valve prolapse (MVP). Her echocardiogram reveals thickened leaflets. You note a grade 3/6 systolic murmur with an ejection click during physical examination. You would advise her that: A) Endocarditis prophylaxis is recommended for most dental and urological procedures B) C) D) Endocarditis prophylaxis is not necessaryShe requires lifetime anticoagulation therapy with warfarin sodium Endocarditis prophylaxis is recommended for dental procedures only

B) Endocarditis prophylaxis is not necessary Prophylaxis treatment for endocarditis is no longer recommended for patients with mitral valve prolapse (MVP).

The span of the normal adult liver is: A) 15 to 18 cm in the midclavicular line B) 6 to 15 cm in midclavicular line C) 2 to 6 cm in the midsternal line D) 4 to 8 cm in the midsternal line

B) 6 to 15 cm in the midclavicular line This range is generally less than 12 cm, but 6 to 15 cm is considered normal for adults.

A positive psoas and obturator sign is highly suggestive of which of the following conditions? A) Ectopic pregnancy B) Acute appendicitis C) PeritonitisD) Abdominal aortic aneurysm

B) Acute appendicitis Both the psoas and obturator signs are associated with acute appendicitis. When the appendix becomes inflamed or ruptured, the blood and pus irritate the psoas and/or obturator muscles, which are both located in the retroperitoneal area. Both muscles are hip flexors and assist with hip movement.

The nurse practitioner would test the obturator and iliopsoas muscle to evaluate for: A) Cholecystitis B) C) D) Acute appendicitis Inguinal hernia Gastric ulcer

B) Acute appendicitis Signs and symptoms of an acute abdomen include involuntary guarding, rebound tenderness, boardlike abdomen, and a positive obturator and psoas sign. A positive obturator sign occurs when pain is elicted by internal rotation of the right hip from 90 degrees hip/knee flexion. The psoas sign is positive when pain occurs with passive extension of the thigh while the patient is lying on his or her side with knees extended, or when pain occurs with active flexion of the thigh at the hip.

Extreme tenderness and involuntary guarding at McBurney's point is a significant finding for possible: A) Acute cholecystitis B) Acute appendicitis C) Acute gastroenteritis D) Acute diverticulitis

B) Acute appendicitis Symptoms of an acute abdomen, such as appendicitis, include extreme tenderness and involuntary guarding at McBurney's point. McBurney's point is the name given to the point over the right side of the abdomen that is one third of the distance (approximately 2 inches) from the anterior superior iliac spine to the umbilicus. This point roughly corresponds to the most common location of the base of the appendix where it is attached to the cecum. (During pregnancy the location of the appendix changes as the uterus grows.)

Which of the following laboratory tests is a sensitive test for evaluating renal function? A) Electrolyte panel B) Estimated glomerular filtration rate (eGFR) C) CreatinineD) Blood urea nitrogen (BUN)

B) Estimated glomerular filtration rate (eGFR) The estimated glomerular filtration rate (GFR) is a sensitive test used to measure and monitor kidney function and evaluate chronic kidney disease (CKD). GFR can be estimated from serum creatinine. The estimated GFR calculation uses serum creatinine along with age and values assigned for gender and race. The National Kidney Foundation has determined different stages of CKD based on the value of estimated GFR.

A homeless 47-year-old man with a history of injection drug use and alcohol abuse presents to the public health clinic with a recent history of fever, night sweats, fatigue, and weakness. The patient has recently noticed some thin red streaks on his nailbed and red bumps on some of his fingers that hurt. During the cardiac exam, the nurse practitioner hears a grade 3/6 murmur over the mitral area. The subcutaneous red-purple nodules are tender to palpation. The thin red lines on the nailbeds resemble subungual splinter hemorrhages. Which of the following conditions is most likely? A) Pericarditis B) C) D) Acute bacterial endocarditis Rheumatic fever Viral cardiomyopathy

B) Acute bacterial endocarditis Bacterial endocarditis is also known as infective endocarditis (IC). It is a serious bacterial infection of the heart valves and the endocardial surface. The bacteria most commonly involved are Staphylococcus and Streptococcus species. Subcutaneous red painful nodules on the finger pads are called Osler's nodes. Subungual splinter hemorrhages on the nailbeds are caused by microemboli. Janeway's lesions are caused by bleeding under the skin (usually located on the palms and the soles) and are painless red papules and macules. Other findings are conjunctival hemorrhages, petechiae, cardiac friction rubs, arrhythmias, murmurs, and others. Three blood cultures obtained at separate sites 1 hour apart are used to identify the causative organism. Some of the risk factors are damaged prosthetic valves, history of rheumatic fever, and injection drug use.

During a breast exam of a 30-year-old nulliparous woman, the nurse practitioner palpates several rubbery mobile areas of breast tissue. They are slightly tender to palpation. Both breasts have symmetrical findings. There are no skin changes or any nipple discharge. The patient is expecting her menstrual period in 5 days. Which of the following would you recommend? A) Referral to a gynecologist for further evaluationB) Advise the patient to return 1 week after her period so her breasts can be recheckedC) Advise the patient to return in 6 months to have her breasts rechecked D) Schedule the patient for a mammogram

B) Advise the patient to return 1 week after her period so her breasts can be rechecked Symptoms of fibrocystic breast disease include cyclic tenderness with prominent breast tissue that is present in both breasts. The symptoms are worse about 1 week before menses. A few days after menses starts, the bloating and breast tenderness resolve. Symptoms are caused by elevated hormone levels (progesterone). Fibrocystic disease is differentiated from breast cancer by the lack of a dominant mass or other symptoms such as peau d'orange, dimpling, retraction, or eczema-like rash on the nipples and areola.

You notice a medium-pitched harsh systolic murmur during an episodic examination of a 37-year-old woman. It is best heard at the right upper border of the sternum. What is most likely? A) Mitral stenosis B) Aortic stenosis C) Pulmonic stenosis D) Tricuspid regurgitation

B) Aortic stenosis One of the most frequent pathological systolic murmurs is due to aortic stenosis. The murmur of aortic stenosis is typically a mid-systolic ejection murmur, heard best over the "aortic area" or right second intercostal space, with radiation into the right neck. It has a harsh quality and may be associated with a palpably slow rise of the carotid upstroke. Additional heart sounds, such as an S4, may be heard secondary to hypertrophy of the left ventricle, which is caused by the greatly increased work required to pump blood through the stenotic valve.

A 56-year-old mechanic is brought to your office complaining of heavy pressure 568 in the substernal area of his chest that is radiating to his jaw. The pain began while he was lifting up a tire. He now appears pale and is diaphoretic. His blood pressure is 100/60 mmHg, and his pulse is 50 beats/min. What is the most appropriate action? A) Perform a 12-lead EKG B) Call 911 C) Administer a morphine injection for pain D) Observe the patient in the office

B) Call 911 Heavy chest pressure in the substernal area radiating to the jaw, diaphoresis, low blood pressure, and bradycardia are signs of cardiac distress. This patient is exhibiting classic symptoms of a myocardial infarction and needs immediate treatment. Call 911 immediately and transfer the patient to the emergency department.

What is the most common cause of left ventricular hypertrophy in the United 544 States? A) Chronic atrial fibrillation B) Chronic hypertension C) Mitral valve prolapse D) Pulmonary hypertension

B) Chronic hypertension Left ventricular hypertrophy develops in response to some factor, such as high blood pressure, that requires the left ventricle to work harder. As the workload increases, the walls of the chamber grow thicker, lose elasticity, and eventually may fail to pump with as much force as a healthy heart. High blood pressure, a blood pressure reading greater than 140/90 mmHg, is the greatest risk factor.

A 68-year-old woman with hypertension and diabetes is seen by the nurse 529 practitioner for a dry cough that worsens at night when she lies in bed. She has shortness of breath, which worsens when she exerts herself, and has gained 6 lbs during the past 2 months. Her pulse rate is 90 beats/min and regular. She is on a nitroglycerine patch and furosemide daily. The best explanation for her symptoms is: A) Kidney failure B) Congestive heart failure C) ACE inhibitor-induced coughing D) Thyroid disease

B) Congestive heart failure In congestive heart failure (CHF), the heart's ventricular function is inadequate. Symptoms include fatigue, diminished exercise capacity, shortness of breath, hemoptysis, cough, orthopnea, hypertension, nocturnal dyspnea, and edema. The kidneys begin to lose their normal ability to excrete sodium and water, leading to fluid retention. Lung congestion/pulmonary edema causes shortness of breath and a decreased ability to tolerate exercise.

Which of the following would you recommend to this 55-year-old patient? A) Start an exercise program with walking instead of jogging B) Consult with a cardiologist for further evaluation C) Consult with a gastroenterologist to rule out acute cholecystitis D) Take ibuprofen (Advil) 600 mg for pain every 4 to 6 hours as needed

B) Consult with a cardiologist for further evaluation Blood tests that indicate tissue damage to the heart include troponin and creatine phosphokinase (CPK). Testing ordered should include EKG, nuclear stress test/stress echocardiogram, and coronary angiography. The patient would need a cardiology consultation for abnormal and/or invasive tests.

A high school teacher complains of a dry cough for the past 6 weeks. It worsens when he is supine. He has episodes of nausea and heartburn, which he self-treats with an over-the-counter (OTC) antacid. He chews mints for his "bad breath." Which of the following is a possible cause for this patient's cough? A) AsthmaB) Gastroesophageal reflux C) PneumoniaD) Chronic postnasal drip

B) Gastroesophageal reflux Classic signs of gastroesophageal reflux disease (GERD) include acid reflux (regurgitation) into the esophagus, heartburn, and nausea. Complications include ulcers, esophageal strictures, Barrett's esophagus, cough, asthma, and throat or laryngeal inflammation. Risk factors include obesity, pregnancy, smoking, and alcohol use.

Which of the following conditions is the most common cause of sudden death 556 among young athletes? A) Brain aneurysm B) Hypertrophic cardiomyopathy C) Left ventricular hypertrophy D) Aortic stenosis

B) Hypertrophic cardiomyopathy Congenital cardiovascular disease is the leading cause of nontraumatic sudden athletic death, with hypertrophic cardiomyopathy being the most common cause. Despite public perception to the contrary, sudden death in young athletes is exceedingly rare. It most commonly occurs in male athletes, who have estimated death rates nearly fivefold greater than the rates of female athletes.

Which of the following conditions is the most common cause of sudden death among young athletes? A) Brain aneurysm B) Hypertrophic cardiomyopathy C) Left ventricular hypertrophy D) Aortic stenosis

B) Hypertrophic cardiomyopathy Congenital cardiovascular disease is the leading cause of nontraumatic sudden athletic death, with hypertrophic cardiomyopathy being the most common cause. Despite public perception to the contrary, sudden death in young athletes is exceedingly rare. It most commonly occurs in male athletes, who have estimated death rates nearly fivefold greater than the rates of female athletes.

A 75-year-old woman presents complaining of a soft lump on her abdomen that is located on the periumbilical area. She tells the nurse practitioner that she does not know how long she has had the lump or whether it has changed in size or shape. She denies abdominal pain, problems with defecation, loss of appetite, weight loss, or trauma. When performing an abdominal exam, what is the best method to differentiate an abdominal wall mass from an intra-abdominal mass? A) Palpate the abdominal wall while the patient is relaxedB) Instruct the patient to lift her head off the table while tensing her abdominal muscles to visualize any masses and then palpate the abdominal wallC) Instruct the patient to lie still for a few seconds while you palpate the abdominal wallD) Palpate the abdomen deeply, then release the palpating hand quickly

B) Instruct the patient to lift her head off the table while tensing her abdominal muscles to visualize any masses and then palpate the abdominal wall An abdominal wall mass will become more prominent when the abdominal wall muscles are tense. If it is an intra-abdominal mass, it will be pressed down by the muscles and will become less obvious or disappear. Some of the most common abdominal wall masses are hernias (epigastric, umbilical, incisional). This patient has a periumbilical hernia (soft lump on her abdomen that is located on the periumbilical area that is painless).

An 8-month-old girl is brought by her grandmother to see the nurse 627 practitioner because of intermittent, random episodes of vomiting, abdominal bloating, currant jelly stools, and irritability with poor appetite. The infant is trending in the 10th percentile on the growth chart and appears lethargic. During the abdominal exam, a sausage-like mass is palpated on the right side of the abdomen. The infant's presentation is highly suggestive of which condition? A) Lactic intoleranceB) IntussusceptionC) Inflammatory bowel disease D) Irritable bowel syndrome

B) Intussusception The "classic triad" of intussusception—currant jelly stools, sausage-like mass, and pain (an infant with inconsolable crying, who draws up knees to abdomen)—is seen in less than 15% of patients. A sausage-shaped abdominal mass may be palpated on the right side of the abdomen. One third of patients do not pass blood or mucus or develop an abdominal mass. Older children may present with pain alone (Kitagawa & Miqdady, 2016).

All of the following statements about phototherapy are correct except: A) Light from the blue-to-white spectrum is used B) It is not always necessary to use a shield for the infant's eyes C) Unconjugated bilirubin in the skin is converted to a water-soluble nontoxic substance that is excreted in the bile D) The infant's eyes should be shielded

B) It is not always necessary to use a shield for the infant's eyes When using phototherapy, the eyes should always be protected by using a shield or goggles to prevent damage to the eyes.

During a routine physical exam of a 90-year-old woman, a low-pitched diastolic murmur grade 2/6 is auscultated. It is located on the fifth intercostal space (ICS) on the left side of the midclavicular line. Which of the following identifications is correct? A) Aortic regurgitation B) Mitral stenosis C) Mitral regurgitation D) Tricuspid regurgitation

B) Mitral stenosis The low-frequency rumbling murmur of mitral stenosis is mid-diastolic and with severity from a short decrescendo murmur to a longer crescendo murmur. It is best heard at the fifth intercostal space, 8 to 9 cm from the midsternal line, slightly medial to the midclavicular line, and does not radiate. Because it is low pitched, it is heard best with the bell of the stethoscope. Aortic regurgitation is a high-pitched diastolic murmur, heard at the second ICS to the right of the sternum. Mitral regurgitation is a pansystolic murmur that radiates to the axilla, and is loud and high pitched when auscultated.

A 40-year-old cashier complains of periods of dizziness and palpitations that have a sudden onset. The EKG shows P waves before each QRS complex and a heart rate of 170 beats/min. A carotid massage decreases the heart rate to 80 beats/min. These findings best describe: A) Ventricular tachycardia B) Paroxysmal atrial tachycardia C) Atrial fibrillation D) Ventricular fibrillation 567

B) Paroxysmal atrial tachycardia Signs and symptoms of paroxysmal atrial tachycardia include a rapid, regular heart rate that begins and ends very quickly. The atria are beating at a very fast rate, but it is not life-threatening. Ventricular tachycardia is usually associated with heart disease, occurs when the ventricles are beating rapidly and inefficiently, and can lead to death if not treated. Atrial fibrillation is an irregular heartbeat that can be life-threatening if not treated. Ventricular fibrillation occurs when the heartbeat is rapid and chaotic, and death will occur if the condition is not treated.

A 40-year-old White woman with a body mass index (BMI) of 32 complains of colicky pain in the right upper quadrant of her abdomen that gets worse if she eats fried food. During the physical exam, the nurse practitioner presses deeply on the left lower quadrant of the abdomen. After she releases her hand, the patient complains of pain on the right side of the lower abdomen. What is the name of this finding? A) Rebound tenderness B) Rovsing's sign C) Murphy's sign D) Psoas test

B) Rovsing's sign The Rovsing sign is right lower quadrant pain intensified by left lower quadrant abdominal pressure (i.e., pain referred to the opposite side of the abdomen after release of palpation). It is associated with peritoneal irritation and appendicitis.

A 70-year-old woman complains of left lower quadrant abdominal pain and fever for 2 days. Her blood pressure of 130/80 mmHg, pulse is 90 beats/min, respirations are 14 breaths/min, and temperature is 100.5°F. During the abdominal exam, the left lower quadrant of the abdomen is tender to palpation. The nurse practitioner (NP) does not palpate a mass; neither is there guarding or rigidity. Rovsing's sign is negative. Bowel sounds are present in all quadrants. The NP is familiar with the patient, who is alert and is asking appropriate questions about her condition. The nurse practitioner suspects that the patient has acute diverticulitis. Which of the following treatment plans is appropriate for this patient? A) The patient should be referred to the physician as soon as possibleB) The patient has a mild case of acute diverticulitis and can be treated with antibiotics in the outpatient setting with close follow-upC) This patient has a moderate to severe case of acute diverticulitis and needs to be admitted to the hospital for IV antibioticsD) The patient should be referred to the emergency department as soon as possible

B) The patient has a mild case of acute diverticulitis and can be treated with antibiotics in the outpatient setting with close follow-up The patient has a mild case of acute diverticulitis and can be treated as an outpatient with antibiotics and a clear fluid diet. If outpatient treatment is selected, close follow-up (within 24-48 hours) is very important. Instruct patients to go to the hospital if symptoms get worse, if fever increases, if unable to tolerate PO treatment, and if pain worsens. Order a complete blood count (to check for leukocytosis, neutrophils, and possible shift to the left), chemistry profile, and urinalysis (to rule out renal causes).

A split S2 heart sound is best heard at which of the following areas? A) The aortic area B) The pulmonic area C) The tricuspid area D) The mitral area

B) The pulmonic area The S2 heart sound is physiologically split in about 688 90% of people. The second heart sound is produced by the closure of the aorticand pulmonic valves. The sound produced by the closure of the aortic valve is termed A2, and the sound produced by the closure of the pulmonic valve is termed P2. The A2 sound is normally much louder than the P2 due to higher pressures in the left side of the heart; thus, A2 radiates to all cardiac listening posts (loudest at the right upper sternal border) and P2 is usually only heard at the left upper sternal border.

While reviewing some lab reports, the nurse practitioner notes that one of the 561 results for her teenage male patient is abnormal. The liver function tests are all normal except for a slight elevation in the alkaline phosphatase level. The patient is a member of a soccer team and denies any recent injury. Which of the following statements is true? A) It is an indication of possible liver damage from alcohol; order a liver ultrasound to rule out fatty liver B) This is a normal finding due to the skeletal growth spurt in this age group C) The patient needs to be evaluated further for pancreatic disease D) The patient needs an ultrasound of the liver to rule out fatty liver and referral to a pediatric rheumatologist

B) This is a normal finding due to the skeletal growth spurt in this age group Alkaline phosphatase is part of a group of related enzymes. The bone form of the enzyme creates the alkaline conditions it requires to be most active with a chemical reaction involving the osteoblasts. The rapid bone growth and increased deposit of calcium during growth spurts and adolescence elevates the alkaline phosphatase level.

When assessing temperature of the skin, which portion of your hand should be used? A) Fingertips B) Palms C) Backs of fingers D) Ulnar aspect of the hand

Backs of fingers The backs of the fingers are thought to be the most temperature sensitive, perhaps because the skin is thinnest there. You may have difficulty detecting subtle differences if you do not use the backs of the fingers.

Which of the following is a clinical identifier of metabolic syndrome? A) Waist circumference of 38 inches for a male B) Waist circumference of 34 inches for a female C) BP of 134/88 for a male D) BP of 128/84 for a female

BP of 134/88 for a male The physical examination criteria for identifying metabolic syndrome include a waist of 40 inches or greater for a male, a waist of 35 inches or greater for a female, and a blood pressure of 130/85 or greater. Other criteria include triglycerides greater than or equal to 150 mg/dL, fasting glucose greater than or equal to 110 mg/dL, and HDL less than 40 for men or less than 50 for women.

When assessing for the femoral pulse, where should the clinician begin deeply palpating?

Below the inguinal ligament, midway between the anterior superior iliac spine & symphysis pubis The external iliac artery transitions into the femoral artery at the level of the inguinal ligament

A 19-year-old female comes to your office, complaining of a clear discharge from her right breast for 2 months. She states that she noticed it when she and her boyfriend were "messing around" and he squeezed her nipple. She continues to have this discharge anytime she squeezes that nipple. She denies any trauma to her breasts. Her past medical history is unremarkable. She denies any pregnancies. Both of her parents are healthy. She denies using tobacco or illegal drugs and drinks three to four beers a week. On examination her breasts are symmetric with no skin changes. You are able to express clear discharge from her right nipple. You feel no discrete masses and her axillae are normal. The remainder of her heart, lung, abdominal, and pelvic examinations are unremarkable. A urine pregnancy test is negative. What cause of nipple discharge is the most likely in her circumstance? A) Benign breast abnormality B) Breast cancer C) Nonpuerperal galactorrhea

Benign breast abnormality Nipple discharge in benign breast abnormalities tends to be clear and unilateral. The discharge is usually not spontaneous. This patient needs to be told to stop compressing her nipple. If the problem still persists after the patient has stopped compressing the nipple, further workup is warranted.

Jim is a 60-year-old man who presents with vomiting. He denies seeing any blood with emesis, which has been occurring for 2 days. He does note a dark, granular substance resembling the coffee left in the filter after brewing. What do you suspect? A) Bleeding from a diverticulum B) Bleeding from a peptic ulcer C) Bleeding from a colon cancer D) Bleeding from cholecystitis

Bleeding from a peptic ulcer When blood is exposed to the environment of the stomach, it often resembles "coffee grounds." This is not always recognized by patients as blood, so it is important to inquire about this. This symptom is not common in cholecystitis, and the other possibilities occur lower in the intestine. It should be noted that conversely, rapid bleeding from the stomach or other upper gastrointestinal source can produce bright red blood in the stool. Do not rule out proximal bleeding on the basis of the absence of "coffee grounds." Likewise, bright red blood seen with emesis may originate from the stomach. Black, sticky stools also can accompany upper GI bleeding.

A 30-year-old man notices a firm, 2-cm mass under his areola. He has no other symptoms and no diagnosis of breast cancer in his first-degree relatives. What is the most likely diagnosis? A) Breast tissue B) Fibrocystic disease C) Breast cancer D) Lymph node

Breast tissue Approximately one third of adult men will have palpable breast tissue under the areola. While males can have breast cancer, this is much less common. There are no lymph nodes in this area.

A 72-year-old retired saleswoman comes to your office, complaining of a bloody discharge from her left breast for 3 months. She denies any trauma to her breast. Her past medical history includes high blood pressure and abdominal surgery for colon cancer. Her aunt died of ovarian cancer and her father died of colon cancer. Her mother died of a stroke. The patient denies tobacco, alcohol, or drug use. She is a widow and has three healthy children. On examination her breasts are symmetric, with no skin changes. You are able to express bloody discharge from her left nipple. You feel no discrete masses, but her left axilla has a hard, 1-cm fixed node. The remainder of her heart, lung, abdominal, and pelvic examinations are unremarkable. What cause of nipple discharge is the most likely in her circumstance? A) Benign breast abnormality B) Breast cancer C) Galactorrhea

Breast CA Nipple discharge in breast cancer is usually unilateral and can be clear or bloody. Although a breast mass is not palpated, in this case a fixed lymph node is palpated. Other forms of breast cancer can present as a chronic rash on the breast.

A 63-year-old nurse comes to your office, upset because she has found an enlarged lymph node under her right arm. She states she found it last week while taking a shower. She isn't sure if she has any breast lumps because she doesn't know how to do self-exams. She states her last mammogram was 5 years ago and it was normal. Her past medical history is significant for high blood pressure and chronic obstructive pulmonary disease. She quit smoking 2 years ago after a 55-packs/year history. She denies using any illegal drugs and drinks alcohol rarely. Her mother died of a heart attack and her father died of a stroke. She has no children. On examination you see an older female appearing her stated age. On visual inspection of her right axilla you see nothing unusual. Palpating this area, you feel a 2-cm hard, fixed lymph node. She denies any tenderness. Visualization of both breasts is normal. Palpation of her left axilla and breast is unremarkable. On palpation of her right breast you feel a nontender 1-cm lump in the tail of Spence. What disorder of the axilla is most likely responsible for her symptoms? A) Breast cancer B) Lymphadenopathy of infectious origin C) Hidradenitis suppurativa

Breast cancer Metastatic lymph nodes tend to be hard, nontender, and fixed, often to the rib cage. Although the patient has no family history of breast cancer, she is at a slightly increased risk due to her never having had children.

What is the best description of Cullen's sign? A) The onset of hyperactive bowel sound before the onset of ileus B) A reddish-purple discoloration that is located on the flank area C) A bluish discoloration or bruising that is located on the umbilical area D) The acute onset of subcutaneous bleeding seen during acute pancreatitis

C) A bluish discoloration or bruising that is located on the umbilical area Cullen's sign is the acute onset of bluish discoloration that is located on the umbilical/periumbilical area, caused by bruising underneath the skin. A bluish discoloration located on the flank area is called the Grey-Turner's sign. It is a sign of a severe case of pancreatitis.

Pulsus paradoxus is best described as: A) An increase in systolic blood pressure on inspiration B) A decrease in diastolic blood pressure on exhalation C) A decrease in systolic blood pressure on inspiration D) An increase in diastolic blood pressure on expiration

C) A decrease in systolic blood pressure on inspiration In patients with pulsus paradoxus, systolic pressure drops on inspiration due to the increased pressure (positive pressure). Some pulmonary risks of having increased pressure include asthma and emphysema.

During a routine physical exam of an 82-year-old woman, the nurse practitioner palpates an irregular mass on the midabdomen that is not tender and is about 2 cm in size. Which of the following is the best initial imaging test to further evaluate the abdominal mass? A) CT scan of the abdomen B) KUB study C) Abdominal ultrasound D) MRI of the abdomen

C) Abdominal ultrasound The ultrasound or sonogram is used as an initial imaging test for abdominal tumors and other types of masses. A CT scan is not considered an initial imaging test in the primary care area. But in many emergency departments, it is used as the initial imaging test in certain cases of abdominal trauma, suspected appendicitis, and other conditions.

While performing a sports physical on a 16-year-old girl, the nurse practitioner notes a split S2 during inspiration that disappears during expiration. The girl is active and her growth and development have been uneventful. What is the best recommendation for the child's mother? A) Recommend referral to a pediatric cardiologist B) Recommend referral for a stress EKG C) Advise the mother that this is a normal finding D) Recommend her daughter avoid strenuous physical exertion until further evaluation

C) Advise the mother that this is a normal finding Explain to the mother that in healthy adolescents, a split S2 during inspiration that disappears during expiration is a normal variation.

When the nurse practitioner is evaluating a patient for intermittent claudication, he or she would first: A) Order a venogram B) Order TED anti-embolism stockings C) Check the ankle and brachial blood pressures before and after exercise D) Check the pedal and posterior tibial pulses

C) Check the ankle and brachial blood pressures before and after exercise Initial evaluation for intermittent claudication would include checking the ankle and brachial blood pressures before and after exercise.

All of the following may help relieve the symptom(s) of gastroesophageal reflux disease (GERD) except: A) Losing weight B) Stopping caffeine intake C) Chewing breath mints D) Stopping alcohol intake

C) Chewing breath mints Gastroesophageal reflux disease (GERD) is a condition in which food comes up the esophagus from the stomach because of a weak sphincter. The reflux is usually worsened by lying down, and can cause a cough and esophageal irritation if not treated. Effective treatment may include weight loss, decreased caffeine intake, and avoidance of alcohol.

While assessing for a cardiac murmur, the first time that a thrill can be palpated is at: A) Grade 2 B) Grade 3 C) Grade 4 D) Grade 5

C) Grade 4 A fine vibration, felt by an examiner's hand on a patient's body over the site of an aneurysm or on the precordium, results from turmoil in the flow of blood and indicates the presence of an organic murmur of grade 4 or greater intensity. A thrill can also be felt over the carotids if a bruit is present and over an arteriovenous fistula in the patient undergoing hemodialysis.

All of the following are correct statements regarding the S3 component of the heart sound except: A) It occurs very early in diastole and is sometimes called an opening snap B) It is a normal finding in some children, healthy young adults, and athletes C) It can be a normal variant if heard in a person aged 40 years or older D) It signifies congestive heart failure (CHF)

C) It can be a normal variant if heard in a person aged 40 years or older The S3 heart sound occurs early in diastole and is sometimes referred to as an opening snap. It is a normal variant in children, healthy young adults, and athletes. Bibasilar crackles in lung bases and the presence of S3 heart sounds are classic findings of congestive heart failure (CHF).

Which of the following is correct regarding the best site to listen for mitral regurgitation? A) It is best heard in the apical area during S2 B) It is best heard at the base during S1 C) It is best heard at the apex during S1 D) It is best heard at the base during S2 569

C) It is best heard at the apex during S1 Mitral regurgitation results from damage to the mitral valve (mitral valve prolapse [MVP], rheumatic heart disease, infective endocarditis). Most patients are initially asymptomatic. The best ausculatory area is the apex or mitral area of the chest. Mitral regurgitation is a systolic (S1) murmur. It may be associated with a midsystolic click if the patient has concurrent MVP. The click is so characteristic of MVP that even without a subsequent murmur, its presence alone is enough for the diagnosis. Immediately after the click, a brief crescendo-decrescendo murmur is heard, usually at the apex. The best imaging test for identifying mitral regurgitation is transthoracic echocardiography.

You note a high-pitched and blowing pansystolic murmur while assessing a 70- year-old male patient. It is grade 2/6 and is best heard at the apical area. Which of the following is most likely? A) Ventricular septal defect B) Tricuspid regurgitation C) Mitral regurgitation D) Mitral stenosis

C) Mitral regurgitation Mitral regurgitation is best heard at the apical area, and manifests as a high-pitched, blowing pansystolic murmur. It occurs when the mitral valve does not close properly. It is the abnormal leaking of blood from the left ventricle, through the mitral valve, and into the left atrium. When the ventricle contracts, there is backflow (regurgitation) of blood into the left atrium. Mitral regurgitation is the most common form of valvular heart disease. Murmurs are graded (classified) depending on how loud they sound with a stethoscope. The scale is 1 to 6 on loudness. A grade 2/6 is a grade 2 on the 6-point scale.

All of the following are true statements about diverticula except: A) Diverticula are located in the colon B) A low-fiber diet is associated with the condition C) Most diverticula in the colon are infected with gram-negative bacteria D) Supplementing with fiber, such as psyllium (Metamucil), is recommended

C) Most diverticula in the colon are infected with gram-negative bacteria Diverticula in the colon can be infected with both gram-negative and gram-positive bacteria.

A 35-year-old man has a history of an upper respiratory viral infection 4 weeks ago. He reports that he started feeling short of breath and now complains of sharp pain in the middle of his chest that seems to worsen when he lies down. The patient's physical exam is within normal limits with the exception of a precordial rub on auscultation. The most likely diagnosis would be: A) Pulmonary embolism B) Dissecting aneurysm C) PericarditisD) Esophageal reflux

C) Pericarditis Pericarditis is inflammation of the sac around the heart. Common signs and symptoms include chest pain over the center/left side of the chest; shortness of breath, especially with lying down; low-grade fever; weakness; fatigue; dry cough; and abdominal or leg swelling. Pericardial rub may be auscultated.

The nurse practitioner orders an ankle-brachial index (ABI) test for a patient. Which of the following disorders is the ABI test used for? A) Venous insufficiency B) Osteoarthritis of the arm or the ankle C) Peripheral arterial disease D) Rheumatoid arthritis

C) Peripheral arterial disease The ankle-brachial index (ABI) is a test that is 702 used to stratify the severity of arterial blockage in the lower extremities for patients with peripheral arterial disease (PAD). An ABI score of 1.0 to 1.4 is normal. Any value less than 1.0 is abnormal. A score of 0.5 or less is indicative of severe PAD.

Which of the following statements is false regarding physiological jaundice in newborns? A) Physiological jaundice is the most common type of jaundice in infants B) The level of unconjugated bilirubin is increased in the newborn's body C) Phototherapy is usually indicated for these infants D) It starts on the second to fourth day of life

C) Phototherapy is usually indicated for these infants The majority of infants with physiological jaundice do not need phototherapy. This form of jaundice is caused by the buildup of unconjugated bilirubin because the infant's immature liver cannot metabolize and excrete it quickly enough.

Which of the following conditions are possible causes of secondary hypertension? A) Leukemia and thalassemia major B) Hashimoto's thyroiditis and polycystic ovaries C) Renal stenosis and adrenal tumors D) Myocardial infarction and coronary artery disease

C) Renal stenosis and adrenal tumors Secondary hypertension is most likely seen following renal stenosis and adrenal tumors. Renal stenosis causes secondary hypertension by plaque formation in the arteries, causing damage to coronary arteries (atherosclerosis). Adrenal tumors initiate secondary hypertension by releasing a large amount of aldosterone, which causes water and salt retention and loss of too much potassium.

Pulsus paradoxus is more likely to be associated with: A) Sarcoidosis B) C) D) Acute bronchitis Status asthmaticus Bacterial pneumonia

C) Status asthmaticus Pulsus paradoxus is most likely to be seen with status asthmaticus. With inspiration, systolic pressure drops due to the increased pressure (positive pressure). Some pulmonary risks of having increased pressure include asthma and emphysema. Cardiac causes for pulsus paradoxus include tamponade, pericarditis, and cardiac effusion.

You suspect an enterobiasis infection in a 6-year-old girl. Which of the following tests would you recommend? A) Stool culture and sensitivity B) Stool for ova and parasites C) The scotch tape test D) Hemoccult test

C) The scotch tape test Enterobiasis infection (pinworms) is caused by small worms that infect the intestines. Symptoms include itching around the anus, which is usually worse at night. The scotch tape test is done by applying the scotch tape on the anal area at bedtime; the worms commonly come out at night and will stick to the tape, which is used for diagnosis.

A 22-year-old sexually active woman is complaining of amenorrhea and new-onset bloody vaginal spotting. On examination, her left adnexa is tender and cervical motion tenderness is positive. Which test should the nurse practitioner order initially? A) Flat plate of the abdomen B) Complete blood count (CBC) with white cell differential C) Urine pregnancy test D) Pelvic ultrasound

C) Urine pregnancy test The patient's history of amenorrhea and new onset of bloody vaginal spotting combined with positive physical findings of left adnexal tenderness and cervical motion tenderness are highly suggestive of an ectopic pregnancy rather than pelvic inflammatory disease (PID). Refer this patient to the emergency department if ectopic pregnancy is suspected. The presence of amenorrhea should be treated as a pregnancy until proven otherwise.

Symptoms suggestive of ulcerative colitis include all of the following except: A) Bloody diarrhea mixed with mucus B) Nausea and vomiting C) Weight gain D) Abdominal pain

C) Weight gain Symptoms of ulcerative colitis include bloody diarrhea mixed with mucus, nausea/vomiting, abdominal pain, and possible weight loss with long- term diarrhea.

A 72-year-old teacher comes to your clinic for an annual examination. She is concerned about her risk for peripheral vascular disease and states that there is a place in town that does tests to let her know her if she has this or not. Which of the following disease processes is a risk factor for peripheral vascular disease? A) Gastroesophageal reflux disease B) Coronary artery disease C) Migraine headaches D) Osteoarthritis

CAD Evidence of coronary artery disease implies that there is most likely disease in other vessels; therefore, this is a risk factor for peripheral vascular disease. Conversely, the presence of peripheral vascular disease is also a risk factor for coronary artery disease, and if present, it should be considered in reduction of cardiac risk factors.

You are assessing a 59-year-old gas station owner for atherosclerosis in the lower extremities. In which of the following locations would the patient's pain make you concerned for this disease process? A) Thigh B) Knee C) Calf D) Ankle

Calf Pain in the calf is the most common site for claudication; however, there could be pain in the buttock, hip, thigh, or foot, depending on the level of the obstruction. The absence of this pain does not rule out significant vascular disease, and actually the minority of these patients are symptomatic.

Which of the following lymph node groups is most commonly involved in breast cancer? A) Lateral B) Subscapular C) Pectoral D) Central

Central The central nodes at the apex of the axilla are most commonly involved in breast cancer. The axilla can be viewed roughly as a four-sided pyramid. An examination covering all sides and the apex is unlikely to miss a significant node.

You are obtaining an arterial blood gas in the radial artery on a retired cab driver who has been hospitalized in the intensive care unit for a stroke. You are concerned about the possibility of arterial insufficiency. You perform the Allen test. This means that you: A) Checked for patency of the radial artery B) Checked for patency of the brachial artery C) Checked for patency of the ulnar artery D) Checked for patency of the femoral artery

Checked for patency of the ulnar artery The Allen test is for determining patency of the ulnar artery before puncturing the radial artery. In the event of an occlusion in the radial artery system, the ulnar artery can provide adequate blood flow.

What is responsible for the inspiratory splitting of S2? A) Closure of aortic, then pulmonic valves B) Closure of mitral, then tricuspid valves C) Closure of aortic, then tricuspid valves D) Closure of mitral, then pulmonic valves

Closure of aortic, then pulmonic valves During inspiration, the closure of the aortic valve and the closure of the pulmonic valve separate slightly, and this may be heard as two audible components, instead of a single sound. Current explanations of inspiratory splitting include increased capacitance in the pulmonary vascular bed during inspiration, which prolongs ejection of blood from the right ventricle, delaying closure of the pulmonic valve. Because the pulmonic component is soft, you may not hear it away from the left second intercostal space. Because it is a low-pitched sound, you may not hear it unless you use the bell of your stethoscope. It is generally easy to hear in school-aged children, and it is easy to notice the respiratory variation of the splitting.

Which of the following events occurs at the start of diastole? A) Closure of the tricuspid valve B) Opening of the pulmonic valve C) Closure of the aortic valve

Closure of the aortic valve At the beginning of diastole, the valves which allow blood to exit the heart close. It is thought that the closure of the aortic valve produces the second heart sound (S2). Closure of the mitral valve is thought to produce the first heart sound (S1).

The S2 heart sound is caused by: A) Closure of the atrioventricular valves B) Closure of the semilunar valves C) Opening of the atrioventricular valves D) Opening of the semilunar valves 512

Closure of the semilunar valves A heart valve normally allows blood to flow in only one direction. A heart valve opens or closes incumbent upon differential blood pressure on each side. A form of heart disease occurs when a valve malfunctions and allows some blood to flow in the wrong direction. The S2 sound results from reverberation within the blood associated with the sudden block of flow reversal.

You would associate a positive iliopsoas muscle test result with which of the following conditions? A) Left cerebral vascular accident B) Urinary tract infection C) Heel fractures D) Acute appendicitis

D) Acute appendicitis A positive iliopsoas muscle test may be seen with acute appendicitis. The right iliopsoas muscle lies under the appendix, This test is performed by asking the patient to actively flex the thigh at the hip. A "positive psoas sign" is noted when the patient exhibits pain in the right lower quadrant due to the inflamed tissue.

All of the following factors have been found to increase the risk of atrial 562 fibrillation in predisposed individuals except: A) Hypertension B) C) D) Excessive alcohol intakeTheophylline (Theo-Dur) and pseudoephedrine (Sudafed) Acute esophagitis

D) Acute esophagitis Factors that influence the risk of atrial fibrillation include hypertension, excessive alcohol consumption, and medications such as Theo-Dur and Sudafed.

Which of the following murmurs can radiate to the neck? A) Mitral stenosis B) Mitral regurgitation C) Aortic regurgitation D) Aortic stenosis

D) Aortic stenosis The murmur of aortic stenosis occurs during systole (S1). The aortic area is in the second intercostal space to the right side of the sternum. The murmur can radiate to the right side of the neck if it is severe.

A 14-year-old boy is brought in by his mother who reports that her son has been complaining for several months of recurrent bloating, stomach upset, and occasional loose stools. She reports that he has difficulty gaining weight and is short for his age. She has noticed that his symptoms are worse after eating large amounts of crackers, cookies, and breads. She denies seeing blood in the boy's stool. Which of the following conditions is most likely? A) AmebiasisB) Malabsorption C) Crohn's colitis D) Celiac disease

D) Celiac disease Celiac disease is also known as celiac sprue. Patients should avoid foods containing gluten, which causes malabsorption (diarrhea, gas, bloating, and abdominal pain). Foods to avoid are wheat, rye, and barley. Oats do not damage the mucosa in celiac disease. Antigliadin IgA and IgG are elevated in almost all patients (90%).

A 65-year-old man with a body mass index (BMI) of 30 and a history of asthma has hypertension that has been well controlled with hydrochlorothiazide 12.5 mg PO daily. His total cholesterol is 230 g/dL. How many risk factors for coronary artery disease (CAD) does he have? A) One risk factor B) Two risk factors C) Three risk factors D) Four risk factors

D) Four risk factors The risk factors for coronary artery disease for this patient are (1) 65-year-old male, (2) overweight (body mass index [BMI] of 30), (3) hypertension, and (4) total cholesterol 230 g/dL.

What type of murmur can radiate to the left axilla? A) Aortic regurgitation B) Aortic stenosis C) Mitral stenosis D) Mitral regurgitation

D) Mitral regurgitation The murmur of mitral regurgitation occurs during systole (holosystolic) and is located in the mitral area of the chest. The location of the mitral area (fifth intercostal space on the left side of the midclavicular line) is near the left axilla, so that a loud murmur can radiate to the left axilla. The causes can be congenital or it may a be sequela of rheumatic fever, mitral valve prolapse, or papillary muscle dysfunction secondary to acute or prior myocardial infarction.

Potential complications of mitral valve prolapse (MVP) include all of the following except: A) Severe mitral regurgitationB) EndocarditisC) Increased risk of stroke and transient ischemic attack D) Mitral stenosis

D) Mitral stenosis Complications of mitral valve prolapse (MVP) include 657 mitral regurgitation, endocarditis, and increased risk of stroke and transient ischemic attacks. The most common complication of mitral valve prolapse (MVP) is mitral valve regurgitation (mitral insufficiency). An abnormal mitral valve increases the chance of developing endocarditis from bacteria, which can further damage the mitral valve. Doctors used to recommend that people with MVP take antibiotics before certain dental or medical procedures to prevent endocarditis (not a current practice). Stroke is a very rare complication of MVP.

A woman who has recently been diagnosed with lupus complains that her hands and feet always feel cold even in the summertime. Sometimes her fingertips become numb and turn a blue color. The fingertips eventually turn dark red in color. Which of the following is most likely? A) Chronic arterial insufficiency B) This is a normal reaction when one feels very cold C) Peripheral vascular disease D) Raynaud's phenomenon

D) Raynaud's phenomenon Raynaud's phenomenon occurs from vasospasms of the blood vessels, leading to decreased blood supply to the hands and feet, which causes bluish discoloration, with fingertips turning a dark-red color if severe. Stress and cold weather are classic triggers for this syndrome.

Your 35-year-old patient is being worked up for microscopic hematuria. All of the following are differential diagnoses of microscopic hematuria except: A) Kidney stones B) Bladder cancer C) Acute pyelonephritis D) Renal artery stenosis

D) Renal artery stenosis Renal artery stenosis refers to narrowing of the kidney arteries. It is commonly noted in individuals older than 50 years of age and is associated with atherosclerosis and hypertension. Hematuria is not associated with renal artery stenosis. Evidence of blood in the urine can be seen with kidney stones, bladder cancer, and acute pyelonephritis.

A 68-year-old woman complains of leaking a small amount of urine whenever she sneezes, laughs, and/or strains. The problem has been present for many months. The patient denies dysuria, frequency, and nocturia. The urine dipstick test is negative for white blood cells, red blood cells, ketones, and urobilinogen. What is the name of this condition? A) Urge incontinence B) Overflow incontinence C) Urinary incontinence D) Stress incontinence

D) Stress incontinence The signs and symptoms of stress incontinence occur when the increased abdominal pressure caused by sneezing, laughing, and/or straining results in the leaking of a small amount of urine through a weakened sphincter. It is more common in younger women (45-49 years old). The urethra and bladder neck fail to close completely due to loss of connective tissue support.

The apex of the heart is located at: 537 A) Second intercostal space to the right of the sternal border B) Second intercostal space to the left of the sternal border C) The left lower sternal border D) The left side of the sternum at the fifth intercostal space by the midclavicular line

D) The left side of the sternum at the fifth intercostal space by the midclavicular line The apex of the heart is directed downward, forward, and to the left. The apex is overlapped by the left lung and pleura. The apex lies behind the fifth left intercostal space, slightly medial to the midclavicular line.

A 74-year-old man presents with recurrent abdominal cramping and pain associated with diarrhea that occurs from four to five times per day. He reports that currently he is having an exacerbation. The stools are bloody with mucus and pus. The patient reports that he has lost weight and is always fatigued. The patient denies recent travel or outdoor camping. Which of the following conditions is most likely? A) GiardiasisB) Irritable bowel syndrome (IBS) C) DiverticulitisD) Ulcerative colitis

D) Ulcerative colitis The most important clue for ulcerative colitis is bloody stools that are covered with mucus and pus along with the systemic symptoms (fatigue, low-grade fever).

73yo retired salesman presents to the ED co CP that started about 2 hours ago. EKG, cardiac enzymes, CXR are normal. RN notes BP in RUE significantly lower than BP in LUE. Based on H&P, which of the following most likely explains s/s?

Dissecting AA Usually have CP, described as "tearing" type pain.

Mr. Patel is a 64-year-old man who was told by another care provider that his liver is enlarged. Although he is a life-long smoker, he has never used drugs or alcohol and has no knowledge of liver disease. Indeed, on examination, a liver edge is palpable 4 centimeters below the costal arch. Which of the following would you do next? A) Check an ultrasound of the liver B) Obtain a hepatitis panel C) Determine liver span by percussion D) Adopt a "watchful waiting" approach

Determine liver span by percussion A liver edge palpable this far below the costal arch should not be ignored. Ultrasound and laboratory investigation are reasonable if the liver is actually enlarged. Mr. Patel has developed emphysema with flattening of the diaphragms. This pushes a normal-sized liver below the costal arch so that it appears to be enlarged. A liver span should be determined by percussing down the chest wall until dullness is heard. A measurement is then made between this point and the lower border of the liver to determine its span; 6-12 centimeters in the mid-clavicular line is normal. Percussion is the only way to assess liver size on examination, and in this case it saved the patient much inconvenience and expense.

76yo retired man with hx of prostate CA & HTN has been screened annually for colon CA using high sensitivity fecal occult blood testing, presents for f/u of HTN during which clinician scans chart to ensure he is up to date with his preventative health care. He has +FOBT on one occasion at age 66yrs, subsequently went for colonoscopy, internal hemorrhoids & sigmoid diverticuli were found on colonoscopy. He has no first-degree relatives with a hx of colorectal CA or adenomatous polyps. What are the USPSTF screening recommendations?

Do not screen routinely

The clinician is palpating pulses in the foot of a diabetic patient while in the clinic, a strong pulse is felt located on the dorsum of the foot, just lateral to the extensor tendon of the big toe. Which artery is being assessed?

Dorsalis pedis The arterial arch of the foot is more distal & runs transversely & is not usually palpable.

32yo cab-driver co pain in LLE, has hx of T2DM, smoker, recently dx with HTN, does not remember injurying his leg, however, notes there is a small wound on lateral aspect of mid-shin. Upon exam, some mild erythema surrounding wound & flat, nonpalpable red streaks progressing up his leg are noted. What do these streaks likely represent?

Draining lymphatic channels Acute lymphangitis is typically caused from an acute bacterial infection of the skin that causes red streaks from distal drainage through the lymphatic system.

Which of the following pairs of ischemic symptoms versus vascular supply is correct? A) Lower calf/superficial femoral B) Erectile dysfunction/iliac or pudendal C) Buttock/common femoral D) Upper calf/tibial or peroneal

ED/iliac or pudendal The ischemia from the iliac or pudendal arteries results in erectile dysfunction. The lower calf is supplied by the popliteal artery, the buttock is supplied by the common femoral artery, and the upper calf is supplied by the superficial femoral artery.

How much does cardiovascular risk increase for each increment of 20 mm Hg systolic and 10 mm Hg diastolic in blood pressure? A) 25% B) 50% C) 75% D) 100%

Each increase of BP by 20 systolic and 10 diastolic doubles the risk of cardiovascular disease. Being "low risk" by JNC 7 criteria confers a 72%-85% reduction in CVD mortality and 40%-58% reduction in overall mortality

Mr. Kruger is an 84-year-old who presents with a smooth lower abdominal mass in the midline which is minimally tender. There is dullness to percussion up to 6 centimeters above the symphysis pubis. What does this most likely represent? A) Sigmoid mass B) Tumor in the abdominal wall C) Hernia D) Enlarged bladder

Enlarged bladder It is possible that this represents a sigmoid colon mass, but this is less likely than an enlarged bladder. Prostatic hypertrophy is very common in this age group and can frequently cause partial urinary obstruction with bladder enlargement. If the mass resolves with catheterization, this is a likely cause. Other forms of urinary obstruction such as neurogenic bladder, urethral stricture, and side effects of drugs can also be contributing to the problem. A hernia would most likely not be dull to percussion. Midline abdominal wall tumors of this size would be unusual but could be discerned by having the patient tense his abdominal muscles.

19yo carwash attendant sustained laceration to ulnar aspect of mid-forearm at work last week. He did not have it evaluated at that time and is now noticing purulent discharge & increasing pain from wound along with fever & chills. Where would clinician expect to find first signs of lymphadenopathy?

Epitrochlear nodes First nodes in drainage region from ulnar surface of forearm & hand, little & ring fingers & adjacent surface of middle finger

A 46-year-old former salesman presents to the ER, complaining of black stools for the past few weeks. His past medical history is significant for cirrhosis. He has gained weight recently, especially around his abdomen. He has smoked two packs of cigarettes a day for 30 years and has drunk approximately 10 alcoholic beverages a day for 25 years. He has used IV heroin and smoked crack in the past. He denies any recent use. He is currently unemployed and has never been married. On examination you find a man appearing older than his stated age. His skin has a yellowish tint and he is thin, with a prominent abdomen. You note multiple "spider angiomas" at the base of his neck. Otherwise, his heart and lung examinations are normal. On inspection he has dilated veins around his umbilicus. Increased bowel sounds are heard during auscultation. Palpation reveals diffuse tenderness that is more severe in the epigastric area. His liver is small and hard to palpation and he has a positive fluid wave. He is positive for occult blood on his rectal examination. What cause of black stools most likely describes his symptoms and signs? A) Infectious diarrhea B) Mallory-Weiss tear C) Esophageal varices

Esophageal varices Varices are often found in alcoholic patients, but only when they have a diagnosis of significant cirrhosis. This patient has symptoms of cirrhosis, including jaundice, ascites, spider hemangiomas, and dilated veins on his abdomen (caput medusa).

In healthy adults over 20, how often should blood pressure, body mass index, waist circumference, and pulse be assessed, according to American Heart Association guidelines? A) Every 6 months B) Every year C) Every 2 years D) Every 5 years

Every 2 years AHA guidelines recommend screening every 2 years in patients over 20 for blood pressure, body mass index, waist circumference, and pulse.

How often, according to American Cancer Society recommendations, should a woman undergo a screening breast examination by a skilled clinician? A) Every year B) Every 2 years C) Every 3 years D) Every 4 years

Every 3 years The current recommendation for screening by breast examination is every 3 years.

You are palpating the abdomen and feel a small mass. Which of the following would you do next? A) Ultrasound B) Examination with the abdominal muscles tensed C) Surgery referral D) Determine size by percussion

Examination with the abdominal muscles tensed It is easy to determine whether the mass is actually in the abdominal wall versus in the abdomen by palpating with the abdominal wall tensed. This can be accomplished by having the patient lift her head off the bed while supine. Usually, abdominal wall masses can be observed, whereas intra-abdominal masses are more concerning.

Which is the most effective pattern of palpation for breast cancer? A) Beginning at the nipple, make an ever-enlarging spiral. B) Divide the breast into quadrants and inspect each systematically. C) Examine in lines resembling the back and forth pattern of mowing a lawn. D) Beginning at the nipple, palpate outward in a stripe pattern.

Examine in lines resembling the back & forth pattern of mowing a lawn The vertical strip pattern has been shown to be the most effective pattern for palpation of the breast. The most important aspect, however, is to be systematic. The tail of Spence, located on the upper anterior chest, is an area commonly missed on examination.

58M with hx of DM & ETOH addiction has been sober for last 10mos, presents with 4mos hx of increasing weakness, recurrent epigastric pain radiating to back, chronic diarrhea with stools 6-8x per day, weight loss of 18lb over 4mos, what is mechanism?

Fibrosis of pancreas Associated with chronic pancreatitis

A 40-year-old mother of two presents to your office for consultation. She is interested in knowing what her relative risks are for developing breast cancer. She is concerned because her sister had unilateral breast cancer 6 years ago at age 38. The patient reports on her history that she began having periods at age 11 and has been fairly regular ever since, except during her two pregnancies. Her first child arrived when she was 26 and her second at age 28. Otherwise she has had no health problems. Her father has high blood pressure. Her mother had unilateral breast cancer in her 70s. The patient denies tobacco, alcohol, or drug use. She is a family law attorney and is married. Her examination is essentially unremarkable. Which risk factor of her personal and family history most puts her in danger of getting breast cancer? A) First-degree relative with premenopausal breast cancer B) Age at menarche of less than 12 C) First live birth between the ages of 25 and 29 D) First-degree relative with postmenopausal breast cancer

First-degree relative with premenopausal breast cancer Having a first-degree relative with cancer before menopause gives a relative risk of 3.1.

54F dietician presents for routine annual exam, on ROS she reports that she had many breast findings over several years, including one biopsy with normal pathology. She feels that her breasts have become far less lumpy since she underwent menopause 3 years ago. Which of the following is true regarding changes in the breasts with menopause?

Glandular tissue of breast atrophies with menopause, primarily due to decrease in number of lobules Consequent decrease in breast density makes mammograms ever more useful during age when breast cancer incidence starts to rise markedly

63yo underweight administrative clerk with 50-pack-year smoking hx presents with several month hx of recurrent epigastric abdominal discomfort. She feels fairly well otherwise & denies nausea, vomiting, diarrhea or constipation. She reports that a first cousin died from a ruptured aneurysm at age 68. Her VS are P 86, BP 148/92, RR 16, O2 95% & T 36.2C. BMI 17.6. On exam abdominal aorta prominent, concerning for AAA. Which is her most significant risk factor?

History of smoking Males > females

A 77-year-old retired nurse has an ulcer on a lower extremity that you are asked to evaluate when you do your weekly rounds at a local long-term care facility. All of the following are responsible for causing ulcers in the lower extremities except for which condition? A) Arterial insufficiency B) Venous insufficiency C) Diminished sensation in pressure points D) Hypertension

HTN Hypertension is not directly associated with the formation of ulcers. It is an indirect risk factor if it is uncontrolled for a long time and associated with atherosclerosis, because it can lead to arterial insufficiency or neuropathy.

A daycare worker presents to your office with jaundice. She denies IV drug use, blood transfusion, and travel and has not been sexually active for the past 10 months. Which type of hepatitis is most likely? A) Hepatitis A B) Hepatitis B C) Hepatitis C D) Hepatitis D

Hepatitis A The lack of contact with blood and body fluids makes hepatitis B, C, and D unlikely. She regularly changes the diapers of her clients and is at risk for hepatitis A. Vaccine against hepatitis A is recommended for daycare workers.

Josh is a 14-year-old boy who presents with a sore throat. On examination, you notice dullness in the last intercostal space in the anterior axillary line on his left side with a deep breath. What does this indicate? A) His spleen is definitely enlarged and further workup is warranted. B) His spleen is possibly enlarged and close attention should be paid to further examination. C) His spleen is possibly enlarged and further workup is warranted. D) His spleen is definitely normal.

His spleen is possibly enlarged & close attention should be paid to further examination This scenario is not uncommon in infectious mononucleosis. The presence of dullness with inspiration should definitely increase your attention to further examination of the spleen, although dullness can occur in normal patients too.

A 42-year-old florist comes to your office, complaining of chronic constipation for the last 6 months. She has had no nausea, vomiting, or diarrhea and no abdominal pain or cramping. She denies any recent illnesses or injuries. She denies any changes to her diet or exercise program. She is on no new medications. During the review of systems you note that she has felt fatigued, had some weight gain, has irregular periods, and has cold intolerance. Her past medical history is significant for one vaginal delivery and two cesarean sections. She is married, has three children, and owns a flower shop. She denies tobacco, alcohol, or drug use. Her mother has type 2 diabetes and her father has coronary artery disease. There is no family history of cancers. On examination she appears her stated age. Her vital signs are normal. Her head, eyes, ears, nose, throat, and neck examinations are normal. Her cardiac, lung, and abdominal examinations are also unremarkable. Her rectal occult blood test is negative. Her deep tendon reflexes are delayed in response to a blow with the hammer, especially the Achilles tendons. What is the best choice for the cause of her constipation? A) Large bowel obstruction B) Irritable bowel syndrome C) Rectal cancer D) Hypothyroidism

Hypothyroidism Many metabolic conditions can interfere with bowel motility. In this case the patient has many symptoms of hypothyroidism, including cold intolerance, weight gain, fatigue, constipation, and irregular menstrual cycles. On examination, thyromegaly and delayed reflexes can help to make the diagnosis. Medication will usually correct these symptoms.

Monique is a 33-year-old administrative assistant who has had intermittent lower abdominal pain approximately one week a month for the past year. It is not related to her menses. She notes relief with defecation, and a change in form and frequency of her bowel movements with these episodes. Which of the following is most likely? A) Colon cancer B) Cholecystitis C) Inflammatory bowel disease D) Irritable bowel syndrome

IBS Although colon cancer should be a consideration, these symptoms are intermittent and no note is made of progression. Cholecystitis usually presents with right upper quadrant pain. Inflammatory bowel disease is often associated with fever and hematochezia. Because there is relief with defecation and there are no mentioned structural or biochemical abnormalities, irritable bowel syndrome seems most likely. This is a very common condition which can be triggered by certain foods and stress.

A 21-year-old receptionist comes to your clinic, complaining of frequent diarrhea. She states that the stools are very loose and there is some cramping beforehand. She states this has occurred on and off since she was in high school. She denies any nausea, vomiting, or blood in her stool. Occasionally she has periods of constipation, but that is rare. She thinks the diarrhea is much worse when she is nervous. Her past medical history is not significant. She is single and a junior in college majoring in accounting. She smokes when she drinks alcohol but denies using any illegal drugs. Both of her parents are healthy. Her entire physical examination is unremarkable. What is most likely the etiology of her diarrhea? A) Secretory infections B) Inflammatory infections C) Irritable bowel syndrome D) Malabsorption syndrome

IBS Irritable bowel syndrome will cause loose bowel movements with cramps but no systemic symptoms of fever, weight loss, or malaise. This syndrome is more likely in young women with alternating symptoms of loose stools and constipation. Stress usually makes the symptoms worse, as do certain foods.

An elderly woman with a history of coronary bypass comes in with severe, diffuse, abdominal pain. Strangely, during your examination, the pain is not made worse by pressing on the abdomen. What do you suspect? A) Malingering B) Neuropathy C) Ischemia D) Physical abuse

Ischemia Ischemic pain can be severe but is not made worse with palpation. The history of bypass could be a clue that there is vascular narrowing elsewhere. Malingering is less likely, and neuropathic pain, as seen in herpes zoster, would worsen with touch. You are to be commended if you considered elder abuse, because this is frequently missed. Ordinarily, this pain would be worse with examination because of the preceding trauma.

Patient has known hx of CVD including MI & +ABI indicating PAD in LLE, now having some issues with ED. Clinician suspects it may be due to meds or further vascular disease. He does not complain of any other symptoms. If his symptoms are related to vascular disease, where would the lesion likely be located?

Iliac pudendal Internal pudendal artery, which is a branch off of the internal iliac artery, is the major blood supply for the penis.

Where is the point of maximal impulse (PMI) normally located? A) In the left 5th intercostal space, 7 to 9 cm lateral to the sternum B) In the left 5th intercostal space, 10 to 12 cm lateral to the sternum C) In the left 5th intercostal space, in the anterior axillary line D) In the left 5th intercostal space, in the midaxillary line

In the left 5th ICS, 7-9cm lateral to sternum The PMI is usually located in the left 5th intercostal space, 7 to 9 centimeters lateral to the sternal border. If it is located more laterally, it usually represents cardiac enlargement. Its size should not be greater than the size of a US quarter, or about an inch. Left ventricular enlargement should be suspected if it is larger. The PMI is often the best place to listen for mitral valve murmurs as well as S3 and S4. The PMI is often difficult to feel in normal patients.

A 14-year-old junior high school student is brought in by his mother and father because he seems to be developing breasts. The mother is upset because she read on the Internet that smoking marijuana leads to breast enlargement in males. The young man adamantly denies using any tobacco, alcohol, or drugs. He has recently noticed changes in his penis, testicles, and pubic hair pattern. Otherwise, his past medical history is unremarkable. His parents are both in good health. He has two older brothers who never had this problem. On examination you see a mildly overweight teenager with enlarged breast tissue that is slightly tender on both sides. Otherwise his examination is normal. He is agreeable to taking a drug test. What is the most likely cause of his gynecomastia? A) Breast cancer B) Imbalance of hormones of puberty C) Drug use

Imbalance of hormones of puberty Approximately one third of teenage boys develop gynecomastia during puberty. It is not surprising that the two older brothers did not have this.

Which is the proper sequence of examination for the abdomen? A) Auscultation, inspection, palpation, percussion B) Inspection, percussion, palpation, auscultation C) Inspection, auscultation, percussion, palpation D) Auscultation, percussion, inspection, palpation

Inspection, auscultation, percussion, palpation The abdominal examination is conducted in a sequence different from other systems, for which the usual order is inspection, percussion, palpation, and auscultation. Because palpation may actually cause some bowel noise when the bowels are not moving, auscultation is performed before percussion and palpation in an abdominal examination.

A 57-year-old maintenance worker comes to your office for evaluation of pain in his legs. He has smoked two packs per day since the age of 16, but he is otherwise healthy. You are concerned that he may have peripheral vascular disease. Which of the following is part of common or concerning symptoms for the peripheral vascular system? A) Intermittent claudication B) Chest pressure with exertion C) Shortness of breath D) Knee pain

Intermittent claudication Intermittent claudication is leg pain that occurs with walking and is relieved by rest. It is a key symptom of peripheral vascular disease. This symptom is present in only about one third of patients with significant arterial disease and, if found, calls for more aggressive management of cardiovascular risk factors. Screening with ankle brachial index can help detect this problem.

A 77-year-old retired bus driver comes to your clinic for a physical examination at his wife's request. He has recently been losing weight and has felt very fatigued. He has had no chest pain, shortness of breath, nausea, vomiting, or fever. His past medical history includes colon cancer, for which he had surgery, and arthritis. He has been married for over 40 years. He denies any tobacco or drug use and has not drunk alcohol in over 40 years. His parents both died of cancer in their 60s. On examination his vital signs are normal. His head, cardiac, and pulmonary examinations are unremarkable. On abdominal examination you hear normal bowel sounds, but when you palpate his liver it is abnormal. His rectal examination is positive for occult blood. What further abnormality of the liver was likely found on examination? A) Smooth, large, nontender liver B) Irregular, large liver C) Smooth, large, tender liver

Irregular, large liver With his past history of colon cancer and with recent weight loss and fatigue, a relapse of his colon cancer would be expected. Colon cancer usually metastasizes to the liver, creating hard, irregular nodules, which can sometimes be palpated on examination. A smooth, large liver which is tender is often seen in hepatitis.

Which of the following is true of a grade 4-intensity murmur? A) It is moderately loud. B) It can be heard with the stethoscope off the chest. C) It can be heard with the stethoscope partially off the chest. D) It is associated with a "thrill."

Is is associated with a "thrill" The grade 4 murmur is differentiated from those below it by the presence of a palpable thrill. A murmur cannot be graded as a 4 unless this is present. The thrill is a "buzzing" feeling over the area where the murmur is loudest. For practice, you may often feel a thrill over a dialysis fistula.

Cody is a teenager with a history of leukemia and an enlarged spleen. Today he presents with fairly significant left upper quadrant pain. On examination of this area a rough grating noise is heard. What is this sound? A) It is a splenic rub. B) It is a variant of bowel noise. C) It represents borborygmi. D) It is a vascular noise.

It is a splenic rub A rough, grating noise over this area represents a splenic rub, which can accompany splenic infarction. Rubs also occur over the liver and pleura and pericardium.

Which is true of splitting of the second heart sound? A) It is best heard over the pulmonic area with the bell of the stethoscope. B) It normally increases with exhalation. C) It is best heard over the apex. D) It does not vary with respiration.

It is best heard over the pulmonic area with the bell of the stethoscope S2 splitting is best heard over the pulmonic area because this is the only place where both of its components can be heard well. The closure of the pulmonic valve is normally not loud because the right heart is a low-pressure system. The bell is best used because it is a low-pitched sound. S2 splitting normally increases with inhalation.

Which is true of a third heart sound (S3)? A) It marks atrial contraction. B) It reflects normal compliance of the left ventricle. C) It is caused by rapid deceleration of blood against the ventricular wall. D) It is not heard in atrial fibrillation.

It is caused by rapid deceleration of blood against the ventricular wall The S3 gallop is caused by rapid deceleration of blood against the ventricular wall. S4 is heard with atrial contraction and is absent in atrial fibrillation for this reason. It usually indicates a stiff or thickened left ventricle as in hypertension or left ventricular hypertrophy.

First year med student examining pt with structurally normal heart, student having difficulty auscultating splitting of 2nd heart sound, at what area would student best hear this?

Left second & third interspace Pulmonic area is left second & third interspace close to sternum

A 62-year-old woman has been followed by you for 3 years and has had recent onset of hypertension. She is still not at goal despite three antihypertensive medicines, and you strongly doubt nonadherence. Her father died of a heart attack at age 58. Today her pressure is 168/94 and pressure on the other arm is similar. What would you do next? A) Add a fourth medicine B) Refer to nephrology C) Get a CT scan D) Listen closely to her abdomen

Listen closely to her abdomen At this point, it is important to consider secondary causes for this woman's hypertension because of its severity, rapidity of progression, and lack of response to therapy. While you will most likely add a fourth medicine, it is important to carefully examine the abdomen for the presence of renal artery bruits. These are usually heard best in the upper quadrants. It may be necessary to have the patient hold her breath, to have a very quiet room, and to listen with the diaphragm for a very soft, high-pitched sound with systole. It may also help to simultaneously feel the patient's pulse (a bruit with both a systolic and diastolic component is very specific for a significant blockage, while a lone systolic bruit may not be abnormal). Obtaining a CT scan is not likely to be useful, and you may save the delay, expense, and inconvenience of a nephrology referral if you can hear a bruit.

You are examining a patient with emphysema in exacerbation and are having difficulty hearing his heart sounds. What should you do to obtain a good examination? A) Listen in the epigastrium. B) Listen to the patient in the left lateral decubitus position. C) Ask the patient to hold his breath for 30 seconds. D) Listen posteriorly.

Listen in the epigastrium It is often difficult to hear the heart well in a patient with emphysema. The shape of the chest as well as the interfering lung noise make examination challenging. By listening in the epigastrium, these barriers can be overcome. It is impractical to ask a patient who is short of breath to hold his breath for a prolonged period. Listening posteriorly would make the heart sounds even softer. It is always a good idea to listen to a patient in the left lateral decubitus position, but in this case it would not make auscultation easier.

63yo janitory hx of adenomatous colonic polyps presents for well visit, basic labs performed to screen for DM & dyslipidemia, electrolytes & liver enzymes are also measured, labs are all normal except for moderate elevations of aspartate and aminotransferase, alanine aminotransferase, glutamyl transferase, alkaline phosphatase as well as mildly elevated total bilirubin. Presents for f/u & assesses liver, which is consistent with hepatomegaly?

Liver palpable 3cm below right costal margin mid clavicular line on expiration Normal on inspiration when liver is pushed down into abdominal cavity, but abnormal on expiration

A 43-year-old store clerk comes to your office upset because she has found an enlarged lymph node under her left arm. She states she found it yesterday when she was feeling pain under her arm during movement. She states the lymph node is about an inch long and is very painful. She checks her breasts monthly and gets a yearly mammogram (her last was 2 months ago), and until now everything has been normal. She states she is so upset because her mother died in her 50s of breast cancer. The patient does not smoke, drink, or use illegal drugs. Her father is in good health. On examination you see a tense female appearing her stated age. On visual inspection of her left axilla you see a tense red area. There is no scarring around the axilla. Palpating this area, you feel a 2-cm tender, movable lymph node underlying hot skin. Other shotty nodes are also in the area. Visualization of both breasts is normal. Palpation of her right axilla and both breasts is unremarkable. Examining her left arm, you see a scabbed-over superficial laceration over her left hand. Upon your questioning, she remembers she cut her hand gardening last week. What disorder of the axilla is most likely responsible for her symptoms? A) Breast cancer B) Lymphadenopathy of infectious origin C) Hidradenitis suppurativa

Lymphadenopathy of infectious origin A lymph node enlarged because of infection is generally hot, tender, and red. Close examination of the skin that drains to that lymph node region is advised. Often there will be a cut or scratch over the involved arm that has an infectious agent. An example is cat scratch disease.

A 25-year-old optical technician comes to your clinic for evaluation of fatigue. As part of your physical examination, you listen to her heart and hear a murmur only at the cardiac apex. Which valve is most likely to be involved, based on the location of the murmur? A) Mitral B) Tricuspid C) Aortic D) Pulmonic

MITRAL Mitral valve sounds are usually heard best at and around the cardiac apex.

In measuring the jugular venous pressure (JVP), which of the following is important? A) Keep the patient's torso at a 45-degree angle. B) Measure the highest visible pressure, usually at end expiration. C) Add the vertical height over the sternal notch to a 5-cm constant. D) Realize that a total value of over 12 cm is abnormal.

Measure the highest visible pressure, usually at end expiration In measuring JVP, the angle of the patient's torso must be varied until the highest oscillation point, or meniscus is visible. This varies. The landmark used is actually the sternal angle, not the sternal notch. We assign a constant height of 5 cm above the right atrium to this landmark. A value of over 8 cm total (more than 3 cm vertical distance above the sternal angle, plus the 5 cm constant) is considered abnormal.

66F museum curator presents for a routine annual examination, on exam, a notably enlarged supraclavicular lymph node is appreciated on the right side. The lymph node is nontender & feels firm and rubbery. She denies any localized or systemic s/s such as breast lumps, fevers, or night sweats. She has been taking conjugated estrogen tablets for 9 years since menopause, though she has not taken progestin compounds since she had a hysterectomy for heavy bleeding at age 45 years. Which of the following is true about this presentation of lymphadenopathy?

Metastatic breast cancer cells may spread directly into the infraclavicular & then supraclavicular nodes without first causing ntoable changes in the axillary nodes. Though axillary lymphadenopathy should be evaluated with age-appropriate imaging to rule out breast CA, cells that are metastasizing from the breasts can pass directly into the infraclavicular, then supraclavicular nodes.

When should a woman conduct breast self-examination with respect to her menses? A) Five to seven days following her menses B) Midcycle C) Immediately prior to menses D) During her menses

Midcycle The breast examination should be conducted during the time with the least estrogen stimulation of the breast tissue. This corresponds to five to seven days following menses.

You are performing a cardiac examination on a patient with shortness of breath and palpitations. You listen to the heart with the patient sitting upright, then have him change to a supine position, and finally have him turn onto his left side in the left lateral decubitus position. Which of the following valvular defects is best heard in this position? A) Aortic B) Pulmonic C) Mitral D) Tricuspid

Mitral The left lateral decubitus position brings the left ventricle closer to the chest wall, allowing mitral valve murmurs to be better heard. If you do not listen to the heart in this position with both the diaphragm and bell in a quiet room, it is possible to miss significant murmurs such as mitral stenosis.

Which valve lesion typically produces a murmur of equal intensity throughout systole? A) Aortic stenosis B) Mitral insufficiency C) Pulmonic stenosis D) Aortic insufficiency

Mitral insufficiency This description fits a holosystolic murmur. Because aortic and pulmonic stenosis murmurs vary with the flow of blood during systole, they typically produce a crescendo-decrescendo murmur. The murmur of aortic insufficiency represents backleak across the valve in diastole. It is a decrescendo pattern murmur, which gets softer as the pressure gradient decreases.

A 21-year-old new mother reports that she has been feeling irritable and jittery almost daily for the past few months. She complains of frequent palpitations and more frequent bowel movements along with weight loss. Her blood pressure is 160/70 mmHg, pulse is 110 beats/min, and she is afebrile. All of the following conditions should be considered in the differential diagnosis for this patient except: A) Mitral regurgitation B) Graves' disease C) Generalized anxiety disorder D) Illicit drug use

Mitral regurgitation Signs and symptoms of mitral regurgitation do not include frequent bowel movements with weight loss.

Suzanne is a 20-year-old college student who complains of chest pain. This is intermittent and is located to the left of her sternum. There are no associated symptoms. On examination, you hear a short, high-pitched sound in systole, followed by a murmur which increases in intensity until S2. This is heard best over the apex. When she squats, this noise moves later in systole along with the murmur. Which of the following is the most likely diagnosis? A) Mitral stenosis B) Mitral insufficiency C) Mitral valve prolapse D) Mitral valve papillary muscle ischemia

Mitral valve prolapse The description above is classic for mitral valve prolapse. The extra sound is a midsystolic click, which is typically a short, high-pitched sound. Mitral stenosis is a soft, low-pitched rumbling murmur which is difficult to hear unless the bell is used in the left lateral decubitus position. Mitral insufficiency is a holosystolic murmur heard best over the apex, and papillary muscle ischemia often creates a mitral insufficiency with its accompanying murmur.

A 40-year-old woman comes to the medical office complaining of palpitations and some light-headedness for the past 6 months. These are random episodes. The nurse practitioner notices a mid-systolic click with a late systolic murmur that is best heard in the apical area during auscultation of the chest. You would suspect: A) Atrial fibrillation B) Sinus arrhythmia C) Mitral stenosis D) Mitral valve prolapse

Mitral valve prolapse Mitral valve prolapse (MVP) occurs when the mitral 647 valve does not close all the way, causing a late systolic murmur heard best in the apical area during auscultation of the chest. Following a normal S1 and briefly quiet systole, the valve suddenly prolapses, resulting in a mid-systolic click. The click is so characteristic of MVP that even without a subsequent murmur, its presence alone is enough for the diagnosis. Immediately after the click, a brief crescendo-decrescendo murmur is heard, usually best at the apex. Symptoms patients may experience at times include palpitations and dizziness.

68yo retired administrative assistant complains of 3mos history of recurring pain after ambulating that radiates from her back in the upper lumbar region into both buttocks, BL thighs & mid-calf regions. Her pain is typically improved by sitting or by leaning forward. The origin of her pain is likely secondary to which of the following?

Neurogenic claudication Can mimic PAD by causing pain related to walking; however it is typically relieved by simply sitting or leaning forward

A 56-year-old female comes to your clinic, complaining of her left breast looking unusual. She says that for 2 months the angle of the nipple has changed direction. She does not do self-examinations, so she doesn't know if she has a lump. She has no history of weight loss, weight gain, fever, or night sweats. Her past medical history is significant for high blood pressure. She smokes two packs of cigarettes a day and has three to four drinks per weekend night. Her paternal aunt died of breast cancer in her forties. Her mother is healthy but her father died of prostate cancer. On examination you find a middle-aged woman appearing older than her stated age. Inspection of her left breast reveals a flattened nipple deviating toward the lateral side. On palpation the nipple feels thickened. Lateral to the areola you palpate a nontender 4-cm mass. The axilla contains several fixed nodes. The right breast and axilla examinations are unremarkable. What visible skin change of the breast does she have? A) Nipple retraction B) Paget's disease C) Peau d'orange sign

Nipple retraction A retracted nipple is flattened or pulled inward or toward the medial, lateral, anterior, or posterior side of the breast. The surrounding skin can be thickened. This is a relatively late finding in breast cancer.

Mr. Martin is a 72-year-old smoker who comes to you for his hypertension visit. You note that with deep palpation you feel a pulsatile mass which is about 4 centimeters in diameter. What should you do next? A) Obtain abdominal ultrasound B) Reassess by examination in 6 months C) Reassess by examination in 3 months D) Refer to a vascular surgeon

Obtain abdominal US A pulsatile mass in this man should be followed up with ultrasound as soon as possible. His risk of aortic rupture is at least 15 times greater if his aorta measures more than 4 centimeters. It would be inappropriate to recheck him at a later time without taking action. Likewise, referral to a vascular surgeon before ultrasound may be premature.

Which of the following is most likely benign on breast examination? A) Dimpling of the skin resembling that of an orange B) One breast larger than the other C) One nipple inverted D) One breast with dimple when the patient leans forward

One breast larger than the other Asymmetry in size of the breasts is a common benign finding. The others are concerning for underlying malignancy.

A 68-year-old mechanic presents to the emergency room for shortness of breath. You are concerned about a cardiac cause and measure his jugular venous pressure (JVP). It is elevated. Which one of the following conditions is a potential cause of elevated JVP? A) Left-sided heart failure B) Mitral stenosis C) Constrictive pericarditis D) Aortic aneurysm

One cause of increased jugular venous pressure is constrictive pericarditis. Others include right-sided heart failure, tricuspid stenosis, and superior vena cava syndrome. You may wish to read about these conditions.

A 58-year-old teacher presents to your clinic with a complaint of breathlessness with activity. The patient has no chronic conditions and does not take any medications, herbs, or supplements. Which of the following symptoms is appropriate to ask about in the cardiovascular review of systems? A) Abdominal pain B) Orthopnea C) Hematochezia D) Tenesmus

Orthopnea Orthopnea, which is dyspnea that occurs when the patient is lying down and improves when the patient sits up, is part of the cardiovascular review of systems and, if positive, may indicate congestive heart failure.

A 15-year-old high school freshman is brought to the clinic by his mother because of chronic diarrhea. The mother states that for the past couple of years her son has had diarrhea after many meals. The patient states that the diarrhea seems the absolute worst after his school lunches. He describes his symptoms as cramping abdominal pain and gas followed by diarrhea. His stools are watery with no specific smell. He denies any nausea, vomiting, constipation, weight loss, or fatigue. He has had no recent illness, injuries, or foreign travel. His past medical history is unremarkable. He denies tobacco, alcohol, or drug use. His parents are both healthy. On examination you see a relaxed young man breathing comfortably. His vital signs are normal and his head, eyes, ears, throat, neck, cardiac, and pulmonary examinations are normal. His abdomen is soft and nondistended. His bowel sounds are active and he has no tenderness, no enlarged organs, and no rebound or guarding. His rectal examination is nontender with no blood on the glove. You collect a stool sample for further study. What is the most likely explanation for this patient's chronic diarrhea? A) Malabsorption syndrome B) Osmotic diarrhea C) Secretory diarrhea

Osmotic diarrhea Usually related to lactose intolerance, watery diarrhea often follows meal ingestion. Crampy abdominal pain, distension, and gas often accompany symptoms. Diarrhea is often provoked by pizza, milkshakes, yogurt, and other lactose-containing foods. This condition is more common in African-Americans, Latinos, Native Americans, and Asians.

55M with OSA has diastolic heart failure, EKG shows significant biatral enlargement, what portion of EKG would be abnormal?

P wave result of atrial depolarization & would therefore have changes associated with atrial enlargement

A 44-year-old female comes to your clinic, complaining of severe dry skin in the area over her right nipple. She denies any trauma to the area. She noticed the skin change during a self-examination 2 months ago. She also admits that she had felt a lump under the nipple but kept putting off making an appointment. She does admit to 6 months of fatigue but no weight loss, weight gain, fever, or night sweats. Her past medical history is significant for hypothyroidism. She does not have a history of eczema or allergies. She denies any tobacco, alcohol, or drug use. On examination you find a middle-aged woman appearing her stated age. Inspection of her right breast reveals a scaly eczema-like crust around her nipple. Underneath you palpate a nontender 2-cm mass. The axilla contains only soft, moveable nodes. The left breast and axilla examination findings are unremarkable. What visible skin change of the breast does she have? A) Nipple retraction B) Paget's disease C) Peau d'orange sign

Paget's disease This uncommon form of breast cancer starts as an eczema-like, scaly skin change around the areola. The lesion may weep, crust, or erode. It can be associated with an underlying mass, but the skin change can also be found alone. Any eczema-like area around the nipple that does not respond to topical treatment needs to be evaluated for breast cancer.

A patient presents with claudication symptoms and diminished pulses. Which of the following is consistent with chronic arterial insufficiency? A) Pallor of the foot when raised to 60 degrees for one minute B) Return of color to the skin within 5 seconds of allowing legs to dangle C) Filling of the veins of the ankles within 10 seconds of allowing the legs to dangle D) Hyperpigmentation of the skin

Pallor of the foot when raised to 60º for 1 minute Pallor of the soles after one minute of elevation is a reliable sign of arterial insufficiency. Return of the color to the skin should occur within 10 seconds of dangling, and the filling of veins should occur within 15 seconds. Hyperpigmentation of the skin is usually seen in venous insufficiency.

You are conducting a workshop on the measurement of jugular venous pulsation. As part of your instruction, you tell the students to make sure that they can distinguish between the jugular venous pulsation and the carotid pulse. Which one of the following characteristics is typical of the carotid pulse? A) Palpable B) Soft, rapid, undulating quality C) Pulsation eliminated by light pressure on the vessel D) Level of pulsation changes with changes in position

Palpable The carotid pulse is palpable; the jugular venous pulsation is rarely palpable. The carotid upstroke is normally brisk, but it may be delayed and decreased as in aortic stenosis or bounding as in aortic insufficiency.

How should you determine whether a murmur is systolic or diastolic? A) Palpate the carotid pulse. B) Palpate the radial pulse. C) Judge the relative length of systole and diastole by auscultation. D) Correlate the murmur with a bedside heart monitor.

Palpate the carotid pulse Timing of a murmur is crucial for identification. The carotid pulse should be used because there is a delay in the radial pulse relative to cardiac events, which can lead to error. Some clinicians can estimate timing by the relative length of systole and diastole, but this method is not reliable at faster heart rates. A bedside monitor is not always available, nor are all designed to correlate in time with the actual pulse.

You are evaluating a 40-year-old banker for coronary heart disease risk factors. He has a history of hypertension, which is well-controlled on his current medications. He does not smoke; he does 45 minutes of aerobic exercise five times weekly. You are calculating his 10-year coronary heart disease risk. Which of the following conditions is considered to be a coronary heart disease risk equivalent?

Peripheral arterial disease Peripheral arterial disease is considered to be a coronary heart disease risk equivalent, as are abdominal aortic aneurysm, carotid atherosclerotic disease, and diabetes mellitus.

A 35F G0P0 presents to clinic with complaint of BL nipple discharge, started several weeks ago, & has occurred at irregular intervals since that time. She does not complain of local tenderness, redness, feveror any other systemic s/s aside from slightly irregular periods over the last few months, on exam, she is able to express a small amount of dc, which is sent to the lab & found to be consistent with breast milk without signs of blood/pus. Screening labs are also sent, which reveal normal blood count, metabolic panel, thyroid-stimulating hormone, HCG. Further labs still pending. What is your most likely diagnosis?

Prolactinoma Pituitary tumors that secrete prolactin, which cause production of breast milk & can suppress menstruation

Elderly pt has hx of smoking 2 packs of cigs/day for 50 yrs, co progressive SOB, on exam, feels most prominent palpable impulse in the xiphoid area, this is most likely result of?

Pulmonary HTN May arise from underlying lung disease such as emphysema or COPD.

61yo retired librarian was recently dx with ovarian CA. She was otherwise healthy until her recent cancer dx. She has not been feeling well lately & has had a cough & some mild shortness of breath for the past couple of days. She now presents to the clinic co pain & swelling in her right groin & leg, which she says is been there for about a week but is worsening. On physical exam, 2+ edema of the right leg up to the thigh; 1+ femoral, popliteal, dorsalis pedis, posterior tibial pulses, no significant erythema are noted. What is the chief concern with this patient?

Pulmonary embolism Cancer pts are at high risk of DVT, and with presenting s/s swelling & pain in her groin, along with recent hx of cough & SOB, this pts presentation is suspicious for PE.

A newborn has an embryologic defect affecting the aortic valve, what other cardiac valve is most likely to be affected?

Pulmonic valve Pulmonic & aortic valves are semilunar valves and have similar embryologic origin

You are performing a thorough cardiac examination. Which of the following chambers of the heart can you assess by palpation? A) Left atrium B) Right atrium C) Right ventricle D) Sinus node

RV The right ventricle occupies most of the anterior cardiac surface and is easily accessible to palpation. The other structures are less likely to have findings on palpation and the sinus node is an intracardiac structure. You may be able to diagnose abnormal rhythms caused by the sinus node indirectly by palpation, but this is less obvious.

A 55-year-old secretary with a recent history of breast cancer, for which she underwent surgery and radiation therapy, and a history of hypertension comes to your office for a routine checkup. Which of the following aspects of the physical are important to note when assessing the patient for peripheral vascular disease in the arms? A) Femoral pulse, popliteal pulse B) Dorsalis pedis pulse, posterior tibial pulse C) Carotid pulse D) Radial pulse, brachial pulse

Radial pulse, brachial pulse This is an important aspect of physical examination to assess for peripheral vascular disease. This patient is at risk for disease in this distribution because of her recent radiation therapy.

68 former paleontologist presents to clinic with concerns about her breast CA risk, her mother developed the disease in her 50s & died from it in her 60s. A younger cousin developed the disease a few years ago before the age of 50yo, but this individual was not tested for BRCA1/2 radiation to the chest. She did take hormone replacement therapy for a few years before data emerged that this may contribute to breast CA risk. She has had several abnormal mammograms in her 50s for persistently dense breasts with subtle findings, but f/u biopsies never showed any malignant pathology. Which of the following is true regarding MRI screening of this pt?

Regardless of recommendations, the high sensitivity of breast MRI comes at the expense of markedly decreased specificity (ability to rule out disease in healthy breasts) Sensitivity & specificity are always a trade-off; that is, a test picks up more true cases is also very likely to then pick up more false positives, and vice versa.

Mr. Edwards complains of cramps and difficulties with walking. The cramps occur in his calves consistently after walking about 100 yards. After a period of rest, he can start to walk again, but after 100 yards these same symptoms recur. Which of the following would suggest spinal stenosis as a cause of this pain? A) Coldness and pallor of the legs B) Relief of the pain with bending at the waist C) Color changes of the skin D) Swelling with tenderness of the skin

Relief of the pain with bending at the waist While these symptoms are classic for claudication, they may also be accounted for by spinal stenosis. Relief with bending at the waist would be consistent with this etiology. Some will state that they must lean over the shopping cart while shopping to avoid these symptoms. Bending stretches the spinal cord and presumably decreases compression. The other symptoms would lead one to suspect a vascular etiology.

70yo retired business executive presents to ED with progressive SOB & 2-pillow orthopnea, on physical exam, BP 145/90, JVD, lower extremity pitting edema to knee, blowing holosystolic murmur heard best at lower left sternal border, no other murmurs or thrills are auscultated on physical exam, which of the following interventions is to most likely improve pts s/s?

Removal of intravascular volume with diuresis HF

As the internal diameter of a blood vessel changes, the resistance changes as well. Which of the following descriptions depicts this relationship? A) Resistance varies linearly with the diameter. B) Resistance varies proportionally to the second power of the diameter. C) Resistance varies proportionally to the third power of the diameter. D) Resistance varies proportionally to the fourth power of the diameter.

Resistance varies proportionally to the fourth power of the diameter The body is able to make significant changes in blood vessel resistance with very small changes to diameter. LaPlace's law tells us that the resistance varies proportionally to the fourth power of the diameter.

Which of the following is consistent with obturator sign? A) Pain distant from the site used to check rebound tenderness B) Right hypogastric pain with the right hip and knee flexed and the hip internally rotated C) Pain with extension of the right thigh while the patient is on her left side or while pressing her knee against your hand with thigh flexion D) Pain that stops inhalation in the right upper quadrant

Right hypogastric pain with the right hip & knee flexed and the hip internally rotated Obturator sign is seen in appendicitis. It is pain with the stretching of the internal obturator muscle because of inflammation. Pain distant from the site used to check rebound tenderness is Rovsing's sign and is a reliable sign of peritonitis. Answer "C" describes psoas sign, which is also seen in appendicitis. Palpation in the right upper quadrant that causes pain severe enough to stop inhalation is consistent with inflammation of the gallbladder and is called Murphy's sign.

24yo graphic designer presents to clinic with concern for breast mass. A rubbery, mobile, nontender mass is palpated in the right breast as described by the pt, which is consistent with a fibroadenoma. In describing the location of the mass, the examiner notes that it is 3cm proximal to & 3cm to the left of the nipple. Which of the following would be the most appropriate way to report this finding?

Rubbery, mobile, nontender mass located in right breast, in the 10:30 position from the nipple. Breast findings can be described by quadrant or by position on a clock face, 12:00 superior edge of breast & nipple at center of clock.

Overweight 26yo public servant presents to ED with 12 hours intense abdominal pain, light-headedness, fainting episode finally prompted her to seek medical attention, has strong family history of gallstones, concerned about that. She has not had any VD, had normal BM this morning, b-HCG is positive. Reports last period was 10 weeks ago, VS P118, BP 86/68, RR 20, O2 99% & T 37.3C oral, palpation of abdomen reveals rigidity and rebound tenderness. DX?

Ruptured tubal (ectopic) pregnancy Constellation of abdominal pain, syncope, tachycardia, hypotension, positive HCG, suggest peritoneal inflammation/irritation strongly suggest ruptured ectopic with significant intra-abdominal bleeding

A toddler with congenital heart disease is seen for a 1-week history of facial and 514 lowerextremity edema accompanied by shortness of breath. The child's mother reports that the child's appetite has been poor. The chest x-ray reveals that the child has congestive heart failure (CHF). Which of the following heart sounds are found in patients with CHF? A) S1 and S2 B) S1, S2, and S3 C) S1, S2, and S4 D) Still's murmur and S4

S1, S2, and S3 Congestive heart failure (CHF) is the inability of the heart to pump a sufficient amount of blood to the organs to meet the body's requirements. It is common to hear S1, S2, and S3 heart sounds on exam. Common signs and symptoms of CHF include fatigue, shortness of breath with activity, and edema of lower extremities.

42F website developer presents for an annual preventative exam with questions about breast CA screening. She is concerned about radiation exposure associated with mammography & interested in MRI as possible alternative for routine screening. She is otherwise healthy with no family hx of breast, ovarian or colon cancer. Which is true about MRI as screening for breast CA in general population?

Sensitivity of screening for breast cancer increases with breast MRI at expense of specificity Increase sensitivity (higher-resolution imaging to pick up subtler disease) is often traded for reduced specificity (in form of discovering many small items of no pathological significance).

A 68-year-old retired waiter comes to your clinic for evaluation of fatigue. You perform a cardiac examination and find that his pulse rate is less than 60. Which of the following conditions could be responsible for this heart rate? A) Second-degree A-V block B) Atrial flutter C) Sinus arrhythmia D) Atrial fibrillation

Second-degree AV block A second-degree A-V block can result in a pulse rate less than 60. Atrial flutter and atrial fibrillation do not cause bradycardia unless there is a significant accompanying block. Sinus arrhythmia does not cause bradycardia and represents respiratory variation of the heart rate.

You notice a patient has a strong pulse and then a weak pulse. This pattern continues. Which of the following is likely? A) Emphysema B) Asthma exacerbation C) Severe left heart failure D) Cardiac tamponade

Severe left heart failure This finding is consistent with pulsus alternans, which is associated with severe left heart failure. Occasionally, a monitor will read only half of the beats because half are too weak to detect. There may also be electrical alternans on EKG. This can be detected by using a blood pressure cuff and lowering the pressure slowly. At one point the rate of Korotkoff sounds will double, because the weaker beats can then "make it through." The other findings are associated with paradoxical pulse.

You are a student in the vascular surgery clinic. You are asked to perform a physical examination on a patient with known peripheral vascular disease in the legs. Which of the following aspects is important to note when you perform your examination? A) Size, symmetry, and skin color B) Muscle bulk and tone C) Nodules in joints D) Lower extremity strength

Size, symmetry and skin color This is an important aspect to note in physical examination. Swelling in the legs, cyanosis, and lack of appropriate hair growth are all signs of peripheral vascular disease.

44yo retail salesperson has noticed an increasing dilatation of veins in her legs, upon inspection, noted that she has significant varicosities on posterior aspects of both legs which begin in the lateral side of the foot & pass upward along the posterior calf. The remainder of the veins in the legs appears normal at this time. Which veins are currently affected?

Small saphenous Great saphenous vein originates on dorsum of foot it passes just anterior to the medial malleolus & continues of the medial aspect of the leg joining the femoral vein of the deep venous system below the inguinal ligament.

A 52-year-old secretary comes to your office, complaining about accidentally leaking urine when she coughs or sneezes. She says this has been going on for about a year now. She relates that she has not had a period for 2 years. She denies any recent illness or injuries. Her past medical history is significant for four spontaneous vaginal deliveries. She is married and has four children. She denies alcohol, tobacco, or drug use. During her pelvic examination you note some atrophic vaginal tissue, but the remainder of her pelvic, abdominal, and rectal examinations are unremarkable. Which type of urinary incontinence does she have? A) Stress incontinence B) Urge incontinence C) Overflow incontinence

Stress incontinence Stress incontinence usually occurs when the intra-abdominal pressure goes up during coughing, sneezing, or laughing. This is usually due to a weakness of the pelvic floor, with inadequate muscle support of the bladder. Vaginal deliveries and pelvic surgery are often associated with these symptoms. Usually female patients are postmenopausal when stress incontinence begins. Kegel exercises are usually recommended to strengthen the pelvic floor muscles.

A patient is concerned about a dark skin lesion on her anterolateral abdomen. It has not changed, and there is no discharge or bleeding. On examination there is a medium brown circular lesion on the anterolateral wall of the abdomen. It is soft, has regular borders, is evenly pigmented, and is about 7 mm in diameter. What is this lesion? A) Melanoma B) Dysplastic nevus C) Supernumerary nipple D) Dermatofibroma

Supernumerary nipple This represents a supernumerary nipple. These occur along the "milk line" and do not exhibit features of more concerning lesions.

65yo farmer who rarely seeks medical care but does have a remote hx of coronary bypass surgery presents to office with 2d hx of increasing SOB & abdominal discomfort. On exam, a protuberant abdomen & LE edema noted. Clinician is concerned about possible RHF & associated ascites and decides to proceed further with physical examination to assess for possible ascites. Which of the following supports this finding?

Tapping on one flank sharply transmits an impulse to the opposite flank Positive fluid wave test finding, but not sensitive or specific to ascites

Which of the following regarding jugular venous pulsations is a systolic phenomenon? A) The "y" descent B) The "x" descent C) The upstroke of the "a" wave D) The downstroke of the "v" wave

The "x" descent The most prominent upstrokes of jugular venous pulsations are diastolic phenomena. These can be timed using the carotid pulse. The only event listed above which is a systolic phenomenon is the "x" descent.

A 27-year-old policewoman comes to your clinic, complaining of severe left-sided back pain radiating down into her groin. It began in the middle of the night and woke her up suddenly. It hurts in her bladder to urinate but she has no burning on the outside. She has had no frequency or urgency with urination but she has seen blood in her urine. She has had nausea with the pain but no vomiting or fever. She denies any other recent illness or injuries. Her past medical history is unremarkable. She denies tobacco or drug use and drinks alcohol rarely. Her mother has high blood pressure and her father is healthy. On examination she looks her stated age and is in obvious pain. She is lying on her left side trying to remain very still. Her cardiac, pulmonary, and abdominal examinations are unremarkable. She has tenderness just inferior to the left costovertebral angle. Her urine pregnancy test is negative and her urine analysis shows red blood cells. What type of urinary tract pain is she most likely to have? A) Kidney pain (from pyelonephritis) B) Ureteral pain (from a kidney stone) C) Musculoskeletal pain D) Ischemic bowel pain

The pain from a kidney stone causes dramatic, severe, colicky pain at the costovertebral angle that radiates across the flank and down into the groin.

A young patient presents with a left-sided mass in her abdomen. You confirm that it is present in the left upper quadrant. Which of the following would support that this represents an enlarged kidney rather than her spleen? A) A palpable "notch" along its edge B) The inability to push your fingers between the mass and the costal margin C) The presence of normal tympany over this area D) The ability to push your fingers medial and deep to the mass

The presence of normal tympany over this area A left upper quadrant mass is more likely to be a kidney if there is no palpable "notch," you can push your fingers between the mass and the costal margin, there is normal tympany over this area, and you cannot push your fingers medial and deep to the mass. These findings are very difficult to appreciate in an obese patient.

Which of the following is true of jugular venous pressure (JVP) measurement?

The vertical height of the blood column in cm, +5cm is the JVP Measurement of the JVP is important to assess a patient's fluid status. Although it may be measured at 45°, it is important to adjust the level of the patient's torso so that the blood column is visible. This may be with the patient completely supine or sitting completely upright, depending on the patient. Any measurement greater than 4 cm above the sternal angle is abnormal. This would correspond to a JVP of 9 cm because we add a constant of 5 cm, which is an estimate of the height of the sternal notch above the right atrium.

Which is true of women who have had a unilateral mastectomy? A) They no longer require breast examination. B) They should be examined carefully along the surgical scar for masses. C) Lymphedema of the ipsilateral arm usually suggests recurrence of breast cancer. D) Women with breast reconstruction over their mastectomy site no longer require examination.

They should be examined carefully along the surgical scar for masses A woman who has had breast cancer remains at high risk for recurrence, especially in the contralateral breast. The mastectomy site should be carefully examined for local recurrence as well. Lymphedema or swelling of the ipsilateral arm following mastectomy is common and does not usually indicate recurrence. Women with breast reconstruction must also undergo careful examination.

You are performing a routine check-up on an 81-year-old retired cotton farmer in the vascular surgery clinic. You note that he has a history of chronic arterial insufficiency. Which of the following physical examination findings in the lower extremities would be expected with this disease? A) Normal pulsation B) Normal temperature C) Marked edema D) Thin, shiny, atrophic skin

Thin, shiny, atrophic skin Thin, shiny, atrophic skin is more commonly seen in chronic arterial insufficiency; in chronic venous insufficiency the skin often has a brown pigmentation and may be thickened.

During a sports physical exam of a 14-year-old high school athlete, the nurse practitioner notices a split of the S2 component of the heart sound during deep inspiration. She notes that it disappears upon expiration. The heart rate is regular and no murmurs are auscultated. Which of the following is correct? A) This is an abnormal finding and should be evaluated further by a cardiologist B) A stress test should be ordered C) This is a normal finding in some young athletes D) An echocardiogram should be ordered

This is a normal finding in some young athletes It is common to hear a split S2 heart sound over the pulmonic area of the heart with inspiration. As long as it disappears with expiration, with no other abnormal symptoms, this is a normal finding. The sound is caused by splitting of the aortic and pulmonic components.

22yo G0P0 undergraduate student presents to clinic after finding breast mass on BSE at home. Mass is nontender without skin changes, erythema or overlying swelling. She has heard that most breast cancers are found by patients themselves, she is very concerned she may have breast cancer. Which of the following is true about BSE & self-detection of breast CA?

This patient is more likely to find a fibroadenoma than a cancer on self-exam. 15-25yo palpable masses are most likely to be benign fibroadenomas

A 48F psychologist presents to clinic with concerns about her breast cancer risk after an age-matched cousin was recently diagnosed with this disease. This cousin is the third family member on her father's side in as many years to be diagnosed with breast cancer, including the patient's own father, who had surgery & subsequent treatment 3 years ago for breast cancer. The pt has little other knowledge of her family hx, only that her grandparents independently arrived from Eastern Europe near the end of WWII & were among very few members of their family that survived the war. The ot has read about testing for BRCA1/2, desires further info about whether this would be appropriate for her. Which of the following is true about BRCA testing?

This pt carries several risk factors that together justify BRCA testing First-degree male relative with breast CA & several other relatives in same lineage with breast CA

44F mathematician presents to clinic with complaint of mass in right breast, partner noticed mass 2 days ago, pt feels guilty bc she has only had 1 mammogram & does not engage in BSE on any regular basis. She has no family hx of breast CA, prior mammogram ordered as routine screening at age 43 after brief discussion with PCP, after thorough investigation reveals benign cyst, what advice should be given to pt about screening for breast CA for her age group?

This pt was in compliance with USPSTF recommendations for her age group & risk factors prior to current complaint. Recommends women <50yo discuss risks & benefits with their provider & decide on appropriate screening for their individual preferences & needs

A clinician, evaluating a pt for valvular competency in the communicating veins of the saphenous system, starts with the pt supine, then elevates one leg to about 90º to empty it of venous blood. Next, the great saphenous vein is the upper part of the thigh is occluded with manual compression & the pt stands. The clinician keeps the vein occluded while watching for venous filling in the leg. Which test is being performed?

Trendelenberg Retrograde filling test, used to evaluate competency of venous valves in the LE

23F comes to respirology clinic for f/u of chronic sinusitis ^ bronchiectasis that is associated with rare congenital condition called Kartagener syndrome. Preceptor notes that she has situs inversus & asks for physical exam. Which of the following descriptions fits with findings on abdominal exam?

Tympany to percussion in the RUQ, dullness to percussion of LUQ Situs inversus rare condition in which organs are reversed & associated with Kartagener syndrome.

Mr. Maxwell has noticed that he is gaining weight and has increasing girth. Which of the following would argue for the presence of ascites? A) Bilateral flank tympany B) Dullness which remains despite change in position C) Dullness centrally when the patient is supine D) Tympany which changes location with patient position

Tympany which changes location with patient position A diagnosis of ascites is supported by findings that are consistent with movement of fluid and gas with changes in position. Gas-filled loops of bowel tend to float so that dullness when supine would argue against this. Likewise, because fluid gathers in dependent areas, the flanks should ordinarily be dull with ascites. Tympany which changes location with patient position ("shifting dullness") would support the presence of ascites. A fluid wave and edema would support this diagnosis as well.

Which area of the arm drains to the epitrochlear nodes? A) Ulnar surface of the forearm and hand, little and ring fingers, and ulnar middle finger B) Radial surface of the forearm and hand, thumb and index fingers, and radial middle finger C) Ulnar surface of the forearm and hand; second, third, and fourth fingers D) Radial surface of the forearm and hand; second, third, and fourth fingers

Ulnar surface of the forearm & hand, little & ring fingers, & ulnar middle finger The epitrochlear node receives lymphatic drainage from the ulnar surface of the forearm and hand, little and ring fingers, and ulnar middle finger. More importantly, it is generally a sign of generalized lymphadenopathy as seen in syphilis and HIV infection.

You are concerned that a patient has an aortic regurgitation murmur. Which is the best position to accentuate the murmur? A) Upright B) Upright, but leaning forward C) Supine D) Left lateral decubitus

Upright, but leaning forward Leaning forward slightly in the upright position brings the aortic valve and the left ventricular outflow tract closer to the chest wall, so it will be easier to hear the soft diastolic decrescendo murmur of aortic insufficiency (regurgitation). You can further your ability to hear this soft murmur by having the patient hold his breath in exhalation.

Linda is a 29-year-old who had excruciating pain which started under her lower ribs on the right side. The pain eventually moved to her lateral abdomen and then into her right lower quadrant. Which is most likely, given this presentation? A) Appendicitis B) Dysmenorrhea C) Ureteral stone D) Ovarian cyst

Ureteral stone The presentation of right flank pain spiraling down to the groin is typical of a ureteral stone. There would most likely be microscopic hematuria as well. The migration pattern of this condition makes the others less likely.

Mrs. LaFarge is a 60-year-old who presents with urinary incontinence. She is unable to get to the bathroom quickly enough when she senses the need to urinate. She has normal mobility. Which of the following is most likely? A) Stress incontinence B) Urge incontinence C) Overflow incontinence D) Functional incontinence

Urge incontinence Stress incontinence occurs with increased intra-abdominal pressure such as with coughing, sneezing, or laughing. This history is most consistent with urge incontinence secondary to detrusor overactivity. Overflow incontinence occurs with anatomic obstruction such as prostatic hypertrophy (obviously not in this case, as the patient is a woman), urethral stricture, or neurogenic bladder. Functional incontinence results from lack of mobility severe enough to impair getting to the bathroom quickly enough.

You are listening carefully for S2 splitting. Which of the following will help? A) Using the diaphragm with light pressure over the 2nd right intercostal space B) Using the bell with light pressure over the 2nd left intercostal space C) Using the diaphragm with firm pressure over the apex D) Using the bell with firm pressure over the lower left sternal border

Using the bell with light pressure over the 2nd left ICS S2 splitting is composed of an aortic and pulmonic component. Because the pulmonic component is softer, it can usually be heard only over the 2nd left intercostal space. It is a low-pitched sound and thus should be sought using the bell with light pressure. Conversely, the diaphragm is best used with firm pressure.

You note a painful ulcerative lesion near the medial malleolus, with accompanying hyperpigmentation. Which of the following etiologies is most likely? A) Arterial insufficiency B) Neuropathic ulcer C) Venous insufficiency D) Trauma

Venous insufficiency These features are most consistent with venous insufficiency. You may also see scaling, redness, varicosities, and other findings. Arterial insufficiency usually affects distal or traumatized areas. Other clues of arterial insufficiency would most likely be present. Neuropathic ulcers occur because of decreased sensation and are common in patients with neuropathy. They are often over bony prominences with surrounding calluses.


Ensembles d'études connexes

Final Exam - Architectural Design

View Set

Español 4 - ALL SPANISH 1-4 VOCAB

View Set

Professional Responsibility Midterm Question Bank

View Set